OB Exam 2 Study Guide

¡Supera tus tareas y exámenes ahora con Quizwiz!

A nurse caring for a family during a loss might notice that a family member is experiencing survivor guilt. Which family member is most likely to exhibit this guilt? A.mother B. Father C. Siblings D. Grandparents

D. Grandparents

The clinic nurse recognizes that the longer an infant is formula fed, the greater is the immunity and resistance the infant will develop against bacterial and viral infections. True False

False

Which nursing action is most appropriate to correct a boggy uterus that is displaced above and to the right of the umbilicus?

Have them go to the bathroom? (R displacement means uRine, L displacement is bLood)

Abruptio placenta is a risk factor for amniotic fluid embolism. True False

True

Endometritis is an infection that usually starts at the placental site. Select one: True False

True

A woman who is 30 weeks of gestation arrives at the hospital with bleeding. Which differential diagnosis would not be applicable for this client? Select one: a. Abruptio placentae b. Spontaneous abortion c. Placenta previa d. Cord insertion

b. Spontaneous abortion

Contraindications for induction of labor include: Select one or more: a. Abnormal fetal position b. Post-term pregnancy c. Pre-eclampsia d. Placental abnormalities

a. Abnormal fetal position d. Placental abnormalities

For which of the following reproductive cancers is high risk HPV infection thought to be primarily responsible? (Select all that apply). Select one or more: a. Cervical cancer b. Ovarian cancer c. Vaginal cancer d. Vulvar cancer e. Endometrial cancer

a. Cervical cancer c. Vaginal cancer d. Vulvar cancer

What is the primary purpose for the use of tocolytic therapy to suppress uterine activity? Select one: a. The most important function of tocolytic therapy is to provide the opportunity to administer antenatal glucocorticoids. b. Drugs can be efficaciously administered up to the designated beginning of term at 37 weeks gestation. c. Tocolytic therapy has no important maternal (as opposed to fetal) contraindications. d. If the client develops pulmonary edema while receiving tocolytic therapy, then intravenous (IV) fluids should be give

a. The most important function of tocolytic therapy is to provide the opportunity to administer

Which of the following lab tests are routinely performed on pregnant women? (Select all that apply). Select one or more: a. 3 hr GTT b. Syphilis test c. Blood type d. Genetic testing for cystic fibrosis e. Rubella titer

b. Syphilis test c. Blood type e. Rubella titer

The nurse suspects that her postpartum client is experiencing hemorrhagic shock. Which observation indicates or would confirm this diagnosis? a.Calm mental status b.Urinary output of at least 30ml/hr c.cool,dry skin d.Absence of cyanosis in the buccal mucosa

b.Urinary output of at least 30ml/hr

A 36 y/o pregnant woman has been diagnosed with polyhydraminos. The nurse knows this is based on which of the following? Select one: a. Amniotic fluid index of 10 cm b. Amniotic fluid index of 30 cm c. Amniotic fluid index of 20 cm d. Amniotic fluid index of 7 cm

c. Amniotic fluid index of 30 cm

A diagnostic test commonly used to assess problems of the fallopian tubes is: Select one: a. Endometrial biopsy b. Ovarian reserve testing c. Hysterosalpingogram d. Screening for sexually transmitted infections

c. Hysterosalpingogram

One of the following neonates is at highest risk for cold stress: Select one: a. LGA neonate at 38 weeks gestation b. AGA neonate at 37 weeks gestation c. SGA neonate at 33 weeks gestation d. SGA neonate at 40 weeks gestation

c. SGA neonate at 33 weeks gestation

Which of the following cancers has no accurate screening test for secondary prevention? Select one: a. Breast cancer b. Uterine cancer c. Cervical cancer d. Ovarian cancer

d. Ovarian cancer

A woman at 28 weeks of gestation experiences blunt abdominal trauma as the result of a fall. The nurse must closely observe the client for what? Select one: a. Alteration in maternal vital signs, especially blood pressure b. Hemorrhage c. Complaints of abdominal pain d. Placental abruption

d. Placental abruption

A woman arrives at the emergency department with complaints of bleeding and cramping. The initial nursing history is significant for a last menstrual period 6 weeks ago. On sterile speculum examination, the primary care provider finds that the cervix is closed. The anticipated plan of care for this woman would be based on a probable diagnosis of which type of spontaneous abortion? Select one: a. Incomplete b. Septic c. Inevitable d. Threatened

d. Threatened

Postoperative care of the pregnant woman who requires abdominal surgery for appendicitis includes which additional assessment? Select one: a. Fetal heart rate (FHR) and uterine activity b. Vital signs and incision c. Intake and output (I&O) and intravenous (IV) site d. Signs and symptoms of infection

a. Fetal heart rate (FHR) and uterine activity

What is the most dangerous effect on the fetus of a mother who smokes cigarettes while pregnant? Select one: a. Intrauterine growth restriction b. Genetic changes and anomalies c. Extensive CNS damage d. Fetal addiction to the substance inhaled

a. Intrauterine growth restriction

To accurately measure the neonate's head, the nurse places the measuring tape around the head: Select one: a. Just above the ears and eyebrows b. Middle of the ear and over the eyes c. Middle of the ear and over the bridge of the nose d. Just below the ears and over the upper lip

a. Just above the ears and eyebrows

Which preexisting factor is known to increase the risk of GDM? Select one: a. Previous birth of large infant b. Maternal age younger than 25 years c. Previous diagnosis of type 2 diabetes mellitus d. Underweight before pregnancy

a. Previous birth of large infant

A couple who has sought infertility counseling has been told that the man's sperm count is very low. The nurse advises the couple that spermatogenesis is impaired when which of the following occur? Select one: a. The testes are overheated. b. The vas deferens is ligated. c. The prostate gland is enlarged. d. alcohol use

a. The testes are overheated.

Which of the following perinatal complications have been associated with untreated asymptomatic bacteriuria? (Select all that apply). Select one or more: a. congenital birth defects b. Low birth weight c. chorioamniotis d. Preterm birth

b. Low birth weight d. Preterm birth

A woman at 28 weeks of gestation experiences blunt abdominal trauma as the result of a fall. The nurse must closely observe the client for what? Select one: a. Alteration in maternal vital signs, especially blood pressure b. Placental abruption c. Hemorrhage d. Complaints of abdominal pain

b. Placental abruption

What is one of the initial signs and symptoms of puerperal infection in the postpartum client? Select one: a. Profuse vaginal lochia with ambulation b. Temperature of 38° C (100.4° F) or higher on 2 successive days c. Pain with voiding d. Fatigue continuing for longer than 1 week

b. Temperature of 38° C (100.4° F) or higher on 2 successive days

What is the most common cause of birth defects in humans? Select one: a. Drugs b. Unknown causes c. Single gene mutations d. Viral infections

b. Unknown causes

Which neurological condition would require preconception counseling, if at all possible? a.Bell Palsy b.Epilepsy c.Eclampsia d.multiple sclerosis

b.Epilepsy

A client is to take Clomiphene Citrate for infertility. Which of the following is the expected action of this medication? Select one: a. Decrease the symptoms of endometriosis b. Increase serum progesterone levels c. Stimulate release of FSH and LH d. Reduce the acidity of vaginal secretions

c. Stimulate release of FSH and LH

The exact cause of preterm labor is unknown but believed to be multifactorial. Infection is thought to be a major factor in many preterm labors. Which type of infection has not been linked to preterm birth? Select one: a. Cervical b. Urinary tract c. Viral d. Periodontal

c. Viral

To explain hemolytic disorders in the newborn to new parents, the nurse who cares for the newborn population must be aware of the physiologic characteristics related to these conditions. What is the most common cause of pathologic hyperbilirubinemia? a.Postmaturity b.Congenital heart defect c.Hemolytic disorders d.Hepatic disease

c.Hemolytic disorders

Combination oral contraceptives, the vaginal ring and the patch contain two hormones. Which of the following is the primary hormone that stops ovulation? Select one: a. hCG b. estrogen c. testosterone d. progesterone

d. progesterone

Hypothyroidism occurs in 2 to 3 pregnancies per 1000. Because severe hypothyroidism is associated with infertility and miscarriage, it is not often seen in pregnancy. Regardless of this fact, the nurse should be aware of the characteristic symptoms of hypothyroidism. Which do they include? (Select all that apply.) Select one or more: a. Decrease in exercise capacity b. Cold intolerance c. Weight loss d. Hot flashes e. Lethargy

a. Decrease in exercise capacity b. Cold intolerance e. Lethargy

Because a full bladder prevents the uterus from contracting normally, nurses intervene to help the woman spontaneously empty her bladder as soon as possible. If all else fails, what tactic might the nurse use? Select one: a. Inserting a sterile catheter b. Asking the physician to prescribe analgesic agents c. Placing oil of peppermint in a bedpan under the woman d. Pouring water from a squeeze bottle over the woman's perineum

a. Inserting a sterile catheter

A woman gave birth to a healthy infant boy 5 days ago. What type of lochia does the nurse expect to find when evaluating this client? Select one: a. Lochia Serosa B. Lochia alba C Lochia Rubra d. Lochia Sangra

a. Lochia Serosa

A serious but uncommon complication of undiagnosed or partially treated hyperthyroidism is a thyroid storm, which may occur in response to stress such as infection, birth, or surgery. What are the signs and symptoms of this emergency disorder? (Select all that apply). Select one or more: a. Restlessness b. Bradycardia c. Hypertension d. Fever e. Hypothermia

a. Restlessness d. Fever

Which nursing intervention is paramount when providing care to a client with preterm labor who has received terbutaline? Select one: a. Assess deep tendon reflexes (DTRs). b. Assess for dyspnea and crackles. c. Assess for bradycardia. d. Assess for hypoglycemia.

b. Assess for dyspnea and crackles.

Which statement is the most appropriate for the nurse to make when caring for bereaved parents? a."You have an angel in heaven." b. "I know how you feel." c. "What can I do for you?" d."This happened for the best."

c. "What can I do for you?"

What is the risk for all pregnancies to have a fetus with minor birth defects? Select one: a. Less than 1% b. 5-9% c. 10-15% d. 3-4%

c. 10-15%

When is a prophylactic cerclage for an incompetent cervix usually placed (in weeks of gestation)? Select one: a. 23 to 24 b. 6 to 8 c. 12 to 14 d. After 24

c. 12 to 14

Having a genetic mutation may create an 85% chance of developing breast cancer in a woman's lifetime. Which condition does not increase a client's risk for breast cancer? Select one: a. BRCA1 or BRCA2 gene mutation b. Cowden syndrome c. Paget disease d. Li-Fraumeni syndrome

c. Paget disease

In evaluating the effectiveness of magnesium sulfate for the treatment of preterm labor, which finding alerts the nurse to possible side effects? Select one: a. Urine output of 160 ml in 4 hours b. Respiratory rate (RR) of 16 breaths per minute c. Serum magnesium level of 10 mg/dl d. DTRs 2+ and no clonus

c. Serum magnesium level of 10 mg/dl

Prostaglandin gel has been ordered for a pregnant woman at 43 weeks of gestation. What is the primary purpose of prostaglandin administration? Select one: a. To enhance uteroplacental perfusion in an aging placenta b. To stimulate the amniotic membranes to rupture c. To ripen the cervix in preparation for labor induction d. To increase amniotic fluid volume

c. To ripen the cervix in preparation for labor induction

A patient, G1 P0, is admitted to the labor and delivery unit for induction of labor. The following assessments were made on admission: Bishop score of 4, fetal heart rate 140s with moderate variability and no decelerations, TPR 98.6°F, 88, 20, BP 120/80, negative obstetrical history. A prostaglandin suppository was inserted at that time. Which of the following findings, 6 hours after insertion, would warrant the removal of the Cervidil (dinoprostone)? Select one: a. Bishop score of 5 b. Fetal heart of 152 bpm c. Respiratory rate of 24 rpm d. More than 5 contractions in 10 minutes

d. More than 5 contractions in 10 minutes

In caring for the preterm infant, what complication is thought to be a result of high arterial blood oxygen level? Select one: a. IVH b. BPD c. NEC d. ROP

d. ROP

Despite warnings, prenatal exposure to alcohol continues to far exceed exposure to illicit drugs. Which condition is rarely associated with fetal alcohol syndrome (FAS)? Select one: a. Neural development disorder b. Alcohol-related birth defects (ARBDs) c. Intellectual impairment d. Respiratory conditions

d. Respiratory conditions

Which newborn reflex is elicited by stroking the lateral sole of the infant's foot from the heel to the ball of the foot? a. Tonic Neck b. Plantar grasp c. Stepping d.Babinski

d.Babinski

Which conditions are infants of diabetic mothers (IDMs) at a higher risk for developing? a.Iron deficiency anemia b.Hyponatremia c.Sepsis d.Respiratory distress syndrome

d.Respiratory distress syndrome

In contrast to placenta previa, what is the most prevalent clinical manifestation of Grade 3 abruptio placentae? Select one: a. Cramping b. Intense abdominal pain c. Bleeding d. Intermittent uterine contractions

b. Intense abdominal pain

Zita is 39 y/o G21002 who is 7 weeks pregnant. For which of the following is she at increased risk? Select one: a. cephalo-pelvic disportion (CPD) b. anemia c.postpartum hemorrhage d.offspring with chromosomal defect

d.offspring with chromosomal defect

Which type of formula is not diluted with water, before being administered to an infant? Select one: a. Ready-to-use b. Powdered c. Modified cow's milk d. Concentrated

a. Ready-to-use

Which information regarding to injuries to the infant's plexus during labor and birth is most accurate? a.Breastfeeding is not recommended for infants with facial nerve paralysis until the condition resolves. b. Erb palsy is damage to the lower plexus. c.If the nerves are stretched with no avulsion, then they should completely recover in 3 to 6 months d.Parents of children with brachial palsy are taught to pick up the child from under the axillae.

c.If the nerves are stretched with no avulsion, then they should completely recover in 3 to 6 months

Which client is most likely to experience strong and uncomfortable afterpains? Select one: a. A woman whose infant weighed 5 pounds, 3 ounces b. A woman who is a gravida 4-4-0-0-4 c. A woman who is bottle-feeding her infant d. A woman who experienced oligohydramnios

b. A woman who is a gravida 4-4-0-0-4

A woman exhibits symptoms that may lead to a possible diagnosis of polycystic ovary syndrome (PCOS). While completing the initial assessment of the client, which clinical finding would the nurse not anticipate? Select one: a. Irregular menses b. Anorexia c. Infertility d. Hirsutism

b. Anorexia

Which intervention by the nurse would reduce the risk of abduction of the newborn from the hospital? Select one: a. Restricting the amount of time infants are out of the nursery b. Applying an electronic and identification bracelet to the mother and the infant c. Instructing the mother not to give her infant to anyone except the one nurse assigned to her that day d. Carrying the infant when transporting him or her in the halls

b. Applying an electronic and identification bracelet to the mother and the infant

An infant boy was delivered minutes ago. The nurse is conducting the initial assessment. Part of the assessment includes the Apgar score. When should the Apgar assessment be performed? a.Every 15 minutes during the newborn's first hour after birth b. At least twice, 1 minute and 5 minutes after birth c.Once by the obstetrician, just after the birth d.Only if the newborn is in obvious distress

b. At least twice, 1 minute and 5 minutes after birth

Preeclampsia begins at what point during the pregnancy? Select one: a. At 12 weeks gestation b. At placental implantation c. After 20 weeks gestation d. At conception

b. At placental implantation

Which statement related to cephalopelvic disproportion (CPD) is the least accurate? Select one: a. The fetus cannot be born vaginally. b. CPD can be accurately predicted c. Causes of CPD may have maternal or fetal origins. d. CPD can be related to either fetal size or fetal position.

b. CPD can be accurately predicted

The nurse is teaching a client with preterm premature rupture of membranes (PPROM) regarding self-care activities. Which activities should the nurse include in her teaching? Select one: a. Tampons are safe to use to absorb the leaking amniotic fluid. b. Do not engage in sexual activity. c. Taking frequent tub baths is safe. d. Report a temperature higher than 40° C.

b. Do not engage in sexual activity.

A woman seeks care at an infertility clinic. Which of the following tests may this woman undergo to determine what, if any, infertility problem she may have? (Select all that apply.) Select one or more: a. Chorionic villus sampling b. Endometrial biopsy c. Hysterosalpingogram d. Serum FSH analysis

b. Endometrial biopsy c. Hysterosalpingogram d. Serum FSH analysis -Common diagnostic tests to determine infertility include: screening for STIs, assessment of hormonal levels (TSH, FSH, LH), assessment for ovulatory dysfunction, detection of LH surge, endometrial biopsy, hysterosalpingogram, and laparoscopy to visualize pelvic structures.. Chorionic Villus Sampling is aspiration of a small amount of placental tissue for genetic testing

Which congenital anomalies can occur as a result of the use of antiepileptic drugs (AEDs) in pregnancy? (Select all that apply.) Select one or more: a. Diaphragmatic hernia b. Neural tube defects c. Cleft lip d. Gastroschisis e. Congenital heart disease

b. Neural tube defects c. Cleft lip e. Congenital heart disease

Which woman has the highest risk for endometrial cancer? Select one: a. Woman who has been on birth control pills for 15 years b. Postmenopausal woman with hypertension c. Perimenopausal woman who has a cystocele d. Woman who has an intrauterine device (IUD)

b. Postmenopausal woman with hypertension

A woman has preinvasive cancer of the cervix. Which modality would the nurse discuss as an available option for a client with this condition? Select one: a. internal radiation b. cryosurgery c. colposcopy d. Hysterectomy

b. cryosurgery

While discussing the societal impacts of breastfeeding, the nurse should be cognizant of the benefits and educate the client accordingly. Which statement as part of this discussion would be incorrect? Select one: a. Breastfeeding costs employers in terms of time lost from work. b. Breastfeeding requires fewer supplies and less cumbersome equipment. c. Breastfeeding saves families money. d. Breastfeeding benefits the environment.

a. Breastfeeding costs employers in terms of time lost from work.

25 year old Susan (G0000) is thinking about getting pregnant this next year, but wants an effective method right now. She currently uses condoms. She denies medical problems. Which of the following birth control methods are most appropriate for the nurse to discuss with Susan? (Select all that apply). Select one or more: a. Condoms (continue) b. The birth control patch c. Oral contraceptive pills d. Copper IUD (Paragard) e. The implant

a. Condoms (continue) b. The birth control patch c. Oral contraceptive pills

Which of the following nursing interventions are important in the prenatal care of the woman with prenatal depression? (Select all of the following). Select one or more: a. Educate the woman about depression and plan of care b. Maintain a caring relationship c. Counsel her on the importance of medication if the woman refuses it. d. Recommend she see a psychiatrist for management

a. Educate the woman about depression and plan of care b. Maintain a caring relationship

The nurse is caring for a postpartum woman who gave birth to a healthy, full-term baby girl. She has a 2-year-old son. She voices concern about her older child's adjustment to the new baby. Nursing actions that will facilitate the older son's adjustment to having a new baby in the house would include which of the following? (Select all that apply.) Select one or more: a. Explain to the mother that she can have her son visit her in the hospital. b. Teach her son how to change the baby's diapers. c. Assist her son in holding his new baby sister. d. Recommend that she spend time reading to her older son while he sits in her lap.

a. Explain to the mother that she can have her son visit her in the hospital. c. Assist her son in holding his new baby sister.

Researchers have found a number of common risk factors that increase a woman's chance of developing a breast malignancy. It is essential for the nurse who provides care to women of any age to be aware of which risk factors? (Select all that apply.) Select one or more: a. Family history b. Early menopause c. Late menarche d. Age e. Nulliparity or first pregnancy after age 40 years

a. Family history d. Age e. Nulliparity or first pregnancy after age 40 years

Documentation related to vacuum delivery includes which of the following: Select one or more: a. Fetal heart rate b. Timing and number of applications c. Position and station of fetal head d. Maternal position

a. Fetal heart rate b. Timing and number of applications c. Position and station of fetal head Assessment of fetal heart rate is part of second-stage management, timing and number of applications are part of standard of care related to safe vacuum deliveries, and position and station of fetal head are noted for safe vacuum extraction. Maternal position is not critical to the documentation related to vacuum deliveries.

Which statement, related to the reconditioning of the urinary system after childbirth, should the nurse understand? Select one: a. Fluid loss through perspiration and increased urinary output accounts for a weight loss of more than 2 kg during the puerperium. b. Kidney function returns to normal a few days after birth. c. With adequate emptying of the bladder, bladder tone is usually restored 2 to 3 weeks after childbirth. d. Diastasis recti abdominis is a common condition that alters the voiding reflex.

a. Fluid loss through perspiration and increased urinary output accounts for a weight loss of more than 2 kg during the puerperium.

In caring for an immediate postpartum client, the nurse notes petechiae and oozing from her intravenous (IV) site. The client would be closely monitored for which clotting disorder? a. Hemorrhage b.Disseminated Intravascular Coagulation (DIC) c.Amniotic fluid embolism (AFE) d.HELLP syndrome

b.Disseminated Intravascular Coagulation (DIC)

What bacterial infection is definitely decreasing in neonates because of effective drug treatment? a. Escherichia coli infection b.Group B streptococci (GBS) infection c. Candidiasis d.Tuberculosis

b.Group B streptococci (GBS) infection

A baby boy was just born to a mother who had positive vaginal cultures for group B streptococci. The mother was admitted to the labor room 30 minutes before the birth. For which of the following should the nursery nurse closely observe this baby? Select one: a. Grunting b. Acrocyanosis c. Pseudostrabismus d. Hydrocele

a. Grunting Group B Strep

Women who are obese are at risk for several complications during pregnancy and birth. Which of these would the nurse anticipate with an obese client? (Select all that apply.) Select one or more: a. Thromboembolism b. Cesarean birth c. Hypertension d. Breech presentation e. Wound infection

a. Thromboembolism b. Cesarean birth c. Hypertension e. Wound infection

The nurse is preparing to administer methotrexate to the client. This drug is most often used for which obstetric complication? Select one: a. Unruptured ectopic pregnancy b. Missed abortion c. Abruptio placentae d. Complete hydatidiform mole

a. Unruptured ectopic pregnancy

When assisting the mother, father, and other family members to actualize the loss of an infant, which action is most helpful? Select one: a. Using the words lost or gone rather than dead or died b. Setting a firm time for ending the visit with the baby so that the parents know when to let go c. Ensuring the baby is clothed or wrapped if the parents choose to visit with the baby d. Making sure the family understands that naming the baby is important

c. Ensuring the baby is clothed or wrapped if the parents choose to visit with the baby

Which classification of placental separation is not recognized as an abnormal adherence pattern? Select one: a. Placenta percreta b. Placenta increta c. Placenta abruptio d. Placenta accreta

c. Placenta abruptio

A primigravida at 40 weeks of gestation is having uterine contractions every 1½ to 2 minutes and states that they are very painful. Her cervix is dilated 2 cm and has not changed in 3 hours. The woman is crying and wants an epidural. What is the likely status of this woman's labor? Select one: a. She is exhibiting hypotonic uterine dysfunction. b. She is experiencing a normal latent stage. c. She is exhibiting hypertonic uterine dysfunction d. She is experiencing precipitous labor.

c. She is exhibiting hypertonic uterine dysfunction

A woman on the day of discharge from the postpartum unit asks a number of questions regarding breastfeeding and shares that she is nervous about taking her baby home and not being able to remember everything she has been taught. These are behaviors associated with: Select one: a. Bonding b. Taking in c. Taking hold d. Attachment

c. Taking hold

What important, immediate postoperative care practice should the nurse remember when caring for a woman who has had a mastectomy? Select one: a. The affected arm should be used for intravenous (IV) therapy. b. Venipuncture for blood work should be performed on the affected arm. c. The blood pressure (BP) cuff should not be applied to the affected arm. d. The affected arm should be held down close to the woman's side

c. The blood pressure (BP) cuff should not be applied to the affected arm.

A recently delivered mother and her baby are at the clinic for a 6-week postpartum checkup. Which response by the client alerts the nurse that psychosocial outcomes have not been met? Select one: a. The woman excessively discusses her labor and birth experience. b. The woman feels that her baby is more attractive and clever than any others. c. The woman has not given the baby a name. d. The woman has a partner or family members who react very positively about the baby.

c. The woman has not given the baby a name.

The nurse is caring for a recently immigrated Chinese woman in the postpartum unit. Based on cultural beliefs and practices of the woman, the nurse would anticipate which of the following? (Select all that apply.) Select one or more: a. The woman prefers cold water for drinking. b. The woman prefers not to shower. c. The woman prefers to have her female relatives care for her baby. d. The woman prefers a wide variety of foods to eat.

c. The woman prefers to have her female relatives care for her baby. In traditional Chinese beliefs and practices, the woman is to rest and female family members take care of the infant. During the first month, the woman is to avoid yin energy by eating specific foods and avoiding drinking or touching cold water. Read p 496

Anna wants to know if her baby has Trisomy 18 as early in the pregnancy as possible. She is 8 weeks pregnant. Her physician will suggest which of the following diagnostic test? a.Noninvasive prenatal screening test b. Amniocentesis c. chorionic villi sampling (CVS) d. sequential integrated test

c. chorionic villi sampling (CVS)

A mother refused to allow her son to receive the vitamin K injection at birth. Which of the following signs or symptoms might the nurse observe in the baby as a result? Select one: a. Skin color is dusky. b. Vital signs are labile. c. Glucose levels are subnormal. d. Circumcision site oozes blood.

d. Circumcision site oozes blood.

The nurse is massaging a boggy uterus. The uterus does not respond to the massage. Which medication would the nurse expect would be given first: Select one: a. Methergine b. Epinephrine c. Carboprost (Hemabate) d. Oxytocin or pitocin

d. Oxytocin or pitocin

Supportive care of the infant with neonatal abstinence syndrome (NAS) include both pharmacologic and nonpharmacologic therapy. Nonpharmacologic therapy would include which of the following (select all that apply): a,Swaddling, clustering care, use of pacifiers to promote "self-soothing" b.Clonidine for infants who do not respond to a single agent c.Use of oral morphine and methadone d.Quiet environment with low lighting and use of soft voice

a,Swaddling, clustering care, use of pacifiers to promote "self-soothing" d.Quiet environment with low lighting and use of soft voice

The perinatal nurse is teaching her new mother about breastfeeding and explains that the most appropriate time to breastfeed is: Select one or more: a. 3 to 4 hours after the last feeding b. When her infant is in a quiet alert state c. When her infant is in an active alert state d. When her infant exhibits hunger-related crying

b. When her infant is in a quiet alert state c. When her infant is in an active alert state

When caring for a pregnant woman with cardiac problems, the nurse must be alert for the signs and symptoms of cardiac decompensation. Which critical findings would the nurse find on assessment of the client experiencing this condition? Select one: a. Shortness of breath, bradycardia, and hypertension b. Regular heart rate and hypertension c. Dyspnea, crackles, and an irregular, weak pulse d. Increased urinary output, tachycardia, and dry cough

c. Dyspnea, crackles, and an irregular, weak pulse

Four babies have just been admitted into the neonatal nursery. Which of the babies should the nurse assess first? a.The baby with respirations 52, oxygen saturation 98% b. The baby with Apgar 9/9, weight 2960 grams c. The baby with temperature 96.3°F, length 17 inches d.The baby with glucose 60 mg/dL, heart rate 132

c. The baby with temperature 96.3°F, length 17 inches

For an infant experiencing symptoms of drug withdrawal, which intervention should be included in the plan of care? a.Feeding every 4 to 6 hours to allow extra rest between feedings b.Playing soft music during feeding c.Snugly swaddling the infant and tightly holding the baby d.Administering chloral hydrate for sedation

c.Snugly swaddling the infant and tightly holding the baby

Which neonatal complications are associated with hypertension in the mother? Select one: a. Placental abruption and DIC b. Seizures and cerebral hemorrhage c. Hepatic or renal dysfunction d. Intrauterine growth restriction (IUGR) and prematurity

d. Intrauterine growth restriction (IUGR) and prematurity

What form of heart disease in women of childbearing years generally has a benign effect on pregnancy? Select one: a. Cardiomyopathy b. Congenital heart disease c. Rheumatic heart disease d. Mitral valve prolapse

d. Mitral valve prolapse

Which hormone remains elevated in the immediate postpartum period of the breastfeeding woman? Select one: a. Estrogen b. Progesterone c. Human placental lactogen d. Prolactin

d. Prolactin

What is the primary rationale for nurses wearing gloves when handling the newborn? Select one: a. Because the nurse has the primary responsibility for the baby during the first 2 hours b. To protect the baby from infection c. As part of the Apgar protocol d. To protect the nurse from contamination by the newborn

d. To protect the nurse from contamination by the newborn

Which condition would require prophylaxis to prevent subacute bacterial endocarditis (SBE) both antepartum and intrapartum? Select one: a. Arrhythmias b. Congestive heart disease c. Postmyocardial infarction d. Valvular heart disease

d. Valvular heart disease

A primigravida has just delivered a healthy infant girl. The nurse is about to administer erythromycin ointment in the infant's eyes when the mother asks, "What is that medicine for?" How should the nurse respond? Select one: a. "Erythromycin is prophylactically given to prevent a gonorrheal infection." b. "This medicine will protect your baby's eyes from drying out over the next few days." c. "It is to protect your baby from contracting herpes from your vaginal tract." d. "It is an eye ointment to help your baby see you better."

a. "Erythromycin is prophylactically given to prevent a gonorrheal infection."

Many first-time parents do not plan on having their parents' help immediately after the newborn arrives. Which statement by the nurse is the most appropriate when counseling new parents regarding the involvement of grandparents? a. "Grandparents can help you with parenting skills." b."Grandparent involvement can be very disruptive to the family." c."You should tell your parents to leave you alone." d. "They are getting old. You should let them be involved while they can."

a. "Grandparents can help you with parenting skills."

A premature infant with respiratory distress syndrome (RDS) receives artificial surfactant. How does the nurse explain surfactant therapy to the parents? Select one: a. "Surfactant improves the ability of your baby's lungs to exchange oxygen and carbon dioxide." b. "Your baby needs this medication to fight a possible respiratory tract infection." c. "The drug keeps your baby from requiring too much sedation." d. "Surfactant is used to reduce episodes of periodic apnea."

a. "Surfactant improves the ability of your baby's lungs to exchange oxygen and carbon dioxide."

The client is being induced in response to worsening preeclampsia. She is also receiving magnesium sulfate. It appears that her labor has not become active, despite several hours of oxytocin administration. She asks the nurse, "Why is this taking so long?" What is the nurse's most appropriate response? Select one: a. "The magnesium is relaxing your uterus and competing with the oxytocin. It may increase the duration of your labor." b. "The length of labor varies for different women." c. "I don't know why it is taking so long." d. "Your baby is just being stubborn."

a. "The magnesium is relaxing your uterus and competing with the oxytocin. It may increase the duration of your labor."

A newborn in the neonatal intensive care unit (NICU) is dying as a result of a massive infection. The parents speak to the neonatologist, who informs them of their son's prognosis. When the father sees his son, he says, "He looks just fine to me. I can't understand what all this is about." What is the most appropriate response or reaction by the nurse at this time? Select one: a. "This must be a difficult time for you. Tell me how you're doing." b. "You'll have to face up to the fact that he is going to die sooner or later." c. "Didn't the physician tell you about your son's problems?" d. Quietly stand beside the infant's father.

a. "This must be a difficult time for you. Tell me how you're doing."

A newborn was admitted to the neonatal intensive care unit (NICU) after being delivered at 29 weeks of gestation to a 28-year-old multiparous, married, Caucasian woman whose pregnancy was uncomplicated until the premature rupture of membranes and preterm birth. The newborn's parents arrive for their first visit after the birth. The parents walk toward the bedside but remain approximately 5 feet away from the bed. What is the nurse's most appropriate action? a. .Go to the parents, introduce him or herself, and gently encourage them to meet their infant. Explain the equipment first, and then focus on the newborn. b.Tell the parents only about the newborn's physical condition and caution them to avoid touching their baby. c.Leave the parents at the bedside while they are visiting so that they have some privacy. d.Wait quietly at the newborn's bedside until the parents come closer.

a. .Go to the parents, introduce him or herself, and gently encourage them to meet their infant. Explain the equipment first, and then focus on the newborn.

A woman with gestational diabetes has had little or no experience reading and interpreting glucose levels. The client shows the nurse her readings for the past few days. Which reading signals the nurse that the client may require an adjustment of insulin or carbohydrates? Select one: a. 50 mg/dl just after waking up from a nap. This is too low; maybe eat a snack before going to sleep. b. 75 mg/dl before lunch. This is low; better eat now. c. 115 mg/dl 1 hour after lunch. This is a little high; maybe eat a little less next time. d. 115 mg/dl 2 hours after lunch. This is too high; it is time for insulin.

a. 50 mg/dl just after waking up from a nap. This is too low; maybe eat a snack before going to sleep.

Which information should nurses provide to expectant mothers when teaching them how to evaluate daily fetal movement counts (DFMCs)? (Select all that apply) Select one or more: a. A count of less than three fetal movements in 1 hour warrants future evaluation. b. Kick counts should be taken every ½ hour and averaged every 6 hours, with every other 6-hour stretch off. c. Alcohol or cigarette smoke can irritate the fetus into greater activity. d. The fetal alarm signal should go off when fetal movements stop entirely for 12 hours

a. A count of less than three fetal movements in 1 hour warrants future evaluation. d. The fetal alarm signal should go off when fetal movements stop entirely for 12 hours

A client is diagnosed with having a stillborn infant. At first, she appears stunned by the news, cries a little, and then asks the nurse to call her mother. What is the proper term for the phase of bereavement that this client is experiencing? Select one: a. Acute distress b. Anticipatory grief c. Reorganization d. Intense grief

a. Acute distress

Postoperative nursing care and education for a woman who had an abdominal hysterectomy includes (select all that apply): Select one or more: a. Administering hormone therapy as per provider orders. b. Assessing vaginal bleeding c. Instructing the woman __vaginal intercourse until she has followup visit with surgeon. d. Instructing the woman to resume activities as comfort level permits.

a. Administering hormone therapy as per provider orders. b. Assessing vaginal bleeding c. Instructing the woman __vaginal intercourse until she has followup visit with surgeon. d. Instructing the woman to resume activities as comfort level permits.

Indications for a primary cesarean birth are often nonrecurring. Therefore, a woman who has had a cesarean birth with a low transverse scar may be a candidate for vaginal birth after cesarean (VBAC). Which clients would be less likely to have a successful VBAC? (Select all that apply.) Select one or more: a. African-American race b. Delivery at a rural hospital c. Maternal obesity (BMI >30) d. Lengthy interpregnancy interval e. Estimated fetal weight <4000 g

a. African-American race b. Delivery at a rural hospital c. Maternal obesity (BMI >30)

Which statement regarding the postpartum uterus is correct? Select one: a. After 2 weeks postpartum, it should be abdominally nonpalpable. b. At the end of the third stage of labor, the postpartum uterus weighs approximately 500 g. c. Postpartum uterus returns to its original (pre-pregnancy) size by 6 weeks postpartum. d. After 2 weeks postpartum, it weighs 100 g.

a. After 2 weeks postpartum, it should be abdominally nonpalpable.

A couple who has been attempting to become pregnant for 5 years is seeking assistance from an infertility clinic. The nurse assesses the clients' emotional responses to their infertility. Which of the following responses would the nurse expect to find? (Select all that apply.) Select one or more: a. Anger at others who have babies. b. Feelings of failure because they cannot make a baby. c. Sexual excitement because they want to conceive a baby. d. Guilt on the part of one partner because he or she is unable to give the other a baby

a. Anger at others who have babies. b. Feelings of failure because they cannot make a baby. d. Guilt on the part of one partner because he or she is unable to give the other a baby -Infertility can be seen as a crisis in the couple's lives and relationship. The diagnosis and treatment of infertility can cause anger with others who have babies, feelings of failure, and feelings of guile. Sexual dysfunction can occur based on the type of infertility testing required and the method of treatment.

According to the American Cancer Society, which of the following is true about screening mammogram? Select one: a. Average-risk women should begin screening mammogram at 45 y/o b. Average-risk women should begin screening mammogram at 40 y/o c. Screening mammograms every year for average-risk women after age 65y/o. d. Only high risk women should get screening mammograms.

a. Average-risk women should begin screening mammogram at 45 y/o

General skin care for full-term infants includes which of the following? (Select all that apply.) Select one or more: a. Avoid daily bathing with soap. b. Use a cleanser with a neutral pH. c. Avoid fragrant soaps. d. Apply petrolatum-based ointments sparingly to dry skin, but avoid head and face.

a. Avoid daily bathing with soap. b. Use a cleanser with a neutral pH. c. Avoid fragrant soaps. d. Apply petrolatum-based ointments sparingly to dry skin, but avoid head and face.

Which of the following is an indication for the administration of methylergonovine (Methergine)? Select one: a. Boggy uterus that does not respond to massage and oxytocin therapy b. Woman with a large hematoma c. Woman with a deep vein thrombosis d. Woman with severe postpartum depression

a. Boggy uterus that does not respond to massage and oxytocin therapy

A nurse is discussing the signs and symptoms of mastitis with a mother who is breastfeeding. Which findings should the nurse include in the discussion? (Select all that apply.) Select one or more: a. Breast tenderness b. Warmth in the breast c. Fever and flulike symptoms d. Small white blister on the tip of the nipple

a. Breast tenderness b. Warmth in the breast c. Fever and flulike symptoms

Which condition might premature infants who exhibit 5 to 10 seconds of respiratory pauses, followed by 10 to 15 seconds of compensatory rapid respiration, be experiencing? Select one: a. Breathing in a respiratory pattern common to premature infants b. Experiencing severe swings in blood pressure c. Trying to maintain a neutral thermal environment d. Suffering from sleep or wakeful apnea

a. Breathing in a respiratory pattern common to premature infants

The nursery nurse notes the presence of diffuse edema on a baby girl's head. Review of the birth record indicates that her mother experienced a prolonged labor and difficult childbirth. By the second day of life, the edema has disappeared. The nurse documents the following condition in the infant's chart. Select one: a. Caput succedaneum b. Cephalhematoma c. Subperiosteal hemorrhage d. Epstein pearls

a. Caput succedaneum

Which statement best describes chronic hypertension? Select one: a. Chronic hypertension can occur independently of or simultaneously with preeclampsia. b. Chronic hypertension is general hypertension plus proteinuria. c. Chronic hypertension is considered severe when the systolic BP is higher than 140 mm Hg or the diastolic BP is higher than 90 mm Hg. d. Chronic hypertension is defined as hypertension that begins during pregnancy and lasts for the duration of the pregnancy.

a. Chronic hypertension can occur independently of or simultaneously with preeclampsia.

A postpartum woman has been diagnosed with postpartum psychosis and will shortly be admitted to the psychiatric unit. Which of the following actions should the nurse perform to ensure safety for both mother and infant? Select one: a. Closely monitor all mother-infant interactions b. Maintain client on strict bed rest. c. Restrict visitation to her partner. d. Carefully monitor toileting.

a. Closely monitor all mother-infant interactions

Near the end of the first week of life, an infant who has not been treated for any infection develops a copper-colored maculopapular rash on the palms and around the mouth and anus. The newborn is displaying signs and symptoms of which condition? Select one: a. Congenital syphilis b. HIV c. Gonorrhea d. Herpes simplex virus (HSV) infection

a. Congenital syphilis

Which of the following tests should never be performed during the first trimester? Select one: a. Contraction Stress test b. Noninvasive Prenatal Screening test c. Maternal screening test for cystic fibrosis d. Ultrasound

a. Contraction Stress test

A primigravida is being monitored at the prenatal clinic for preeclampsia. Which finding is of greatest concern to the nurse? Select one: a. Dipstick value of 3+ for protein in her urine b. Blood pressure (BP) increase to 138/86 mm Hg c. Weight gain of 0.5 kg during the past 2 weeks d. Pitting pedal edema at the end of the day

a. Dipstick value of 3+ for protein in her urine

A first-time mother informs her nurse that she is concerned about infant abduction. The nurse should explain to the parents which of the following? (Select all that apply.) Select one or more: a. Do not allow a person without proper unit specific hospital ID to take their baby. b. Encourage parents to accompany any person who removes their infant from the hospital room c. Instruct parents not to leave their newborn unattended at any time d. Inform parents that ID bands with matching identification numbers are placed on the parents and infant at birth to ensure identification of the correct infant with the correct parents

a. Do not allow a person without proper unit specific hospital ID to take their baby. b. Encourage parents to accompany any person who removes their infant from the hospital room c. Instruct parents not to leave their newborn unattended at any time d. Inform parents that ID bands with matching identification numbers are placed on the parents and infant at birth to ensure identification of the correct infant with the correct parents

To manage her diabetes appropriately and to ensure a good fetal outcome, how would the pregnant woman with diabetes alter her diet? Select one: a. Eat her meals and snacks on a fixed schedule. b. Increase her consumption of protein. c. Eat six small equal meals per day. d. Reduce the carbohydrates in her diet.

a. Eat her meals and snacks on a fixed schedule.

16 y/o Susie presents to the clinic, wanting a pregnancy test. What questions would be appropriate for the professional nurse to ask Susie prior to completing the pregnancy test? Select one or more: a. Have you been using birth control? b. Are you excited that you might be pregnant? c. What is the first day of your last menstrual period? d. Does your mother know you are wanting a pregnancy test? e. Have you done a home pregnancy test?

a. Have you been using birth control? c. What is the first day of your last menstrual period? e. Have you done a home pregnancy test?

The labor of a pregnant woman with preeclampsia is going to be induced. Before initiating the oxytocin (Pitocin) infusion, the nurse reviews the woman's latest laboratory test findings, which reveal a platelet count of 90,000 mm3, an elevated aspartate aminotransaminase (AST) level, and a falling hematocrit. The laboratory results are indicative of which condition? Select one: a. Hemolysis, elevated liver enzyme levels, and low platelet levels (HELLP) syndrome b. Disseminated intravascular coagulation (DIC) syndrome c. Idiopathic thrombocytopenia d. Eclampsia

a. Hemolysis, elevated liver enzyme levels, and low platelet levels (HELLP) syndrome

Infants born between 34 0/7 and 36 6/7 weeks of gestation are called late-preterm infants because they have many needs similar to those of preterm infants. Because they are more stable than early-preterm infants, they may receive care that is similar to that of a full-term baby. These infants are at increased risk for which conditions? (Select all that apply.) Select one or more: a. Hyperbilirubinemia b. Sepsis c. Hyperglycemia d. Cardiac distress e. Problems with thermoregulation

a. Hyperbilirubinemia b. Sepsis e. Problems with thermoregulation

The nurse is teaching new parents about metabolic screening for the newborn. Which statement is most helpful to these clients? Select one: a. If genetic screening is performed before the infant is 24 hours old, then it should be repeated at age 1 to 2 weeks. b. Federal law prohibits newborn genetic testing without parental consent. c. Hearing screening is now mandated by federal law. d. All states test for phenylketonuria (PKU), hypothyroidism, cystic fibrosis, and sickle cell diseases.

a. If genetic screening is performed before the infant is 24 hours old, then it should be repeated at age 1 to 2 weeks.

Mary G10000 is 12 weeks pregnant in October. She is not sure if she wants the flu shot. What is the best message the nurse should include in their education for Mary? Select one: a. If pregnant women get the flu, they can get seriously ill, require hospitalization and possibly die. b. If pregnant women get the flu, they can get sick, require hospitalization and possibly die. It is really up to you to get the flu shot. c. If pregnant women get the flu, they get a little ill. It is OK if you choose not to get flu shot even if it is safe. d. If pregnant women get the flu, they usually don't get seriously ill. The flu shot is a good idea for women to get.

a. If pregnant women get the flu, they can get seriously ill, require hospitalization and possibly die.

Which infant is most likely to express Rh incompatibility? Select one: a. Infant of an Rh-negative mother and a father who is Rh positive and homozygous for the Rh factor b. Infant who is Rh negative and a mother who is Rh negative c. Infant of an Rh-negative mother and a father who is Rh positive and heterozygous for the Rh factor d. Infant who is Rh positive and a mother who is Rh positive

a. Infant of an Rh-negative mother and a father who is Rh positive and homozygous for the Rh factor

A woman gave birth to a 3200 g baby girl with an estimated gestational age of 40 weeks. The baby is 1 hour of age. In preparation for administration of Vitamin K to the infant, the nurse will explain to the parents that an injection of this medication: Select one: a. Influences the activation of coagulation factors to prevent delayed clotting and hemorrhagic disease b. Prevents high levels of unconjugated bilirubin in the newborn's blood c. Prevents the excessive loss of RBCs d. Aids the liver in regulation of blood glucose

a. Influences the activation of coagulation factors to prevent delayed clotting and hemorrhagic disease

A mother of a 10-day-old infant calls the clinic and reports that her baby is having loose, green stools. The mother is breastfeeding her infant. Which of the following is the best nursing action? Select one: a. Instruct the woman to bring her infant to the clinic. b. Instruct the woman to decrease the amount of feeding for 24 hours and to call if the stools continue to be loose. c. Explain that this is a normal stool pattern. d. Instruct the woman to eat a bland diet for the next 24 hours and call back if the stools continue to be loose and green.

a. Instruct the woman to bring her infant to the clinic. (Instruct parents to notify the health care provider if stools are runny and green and/or if newborn/infant has less than 6 wet diapers per day.)

The nurse is working with a 36-year-old, married client, with 6 children who smokes. The woman states, "I don't expect to have any more kids, but I hate the thought of being sterile." Which of the following contraceptive methods would be best for the nurse to recommend to this client? Select one: a. Intrauterine contraceptive device (IUD) b. Contraceptive patch c. Bilateral tubal ligation d. Birth control pills with estrogen and progestin

a. Intrauterine contraceptive device (IUD)

Augmentation of labor: Select one: a. Is part of the active management of labor instituted when the labor process is unsatisfactory and uterine contractions are ineffective b. Relies on more invasive methods when oxytocin and amniotomy have failed c. Is elective induction of labor d. Is an operative vaginal delivery that uses vacuum cups Labor augmentation is the stimulation of ineffective UCs after the onset of spontaneous labor to manage labor dystocia. Lower doses of oxytocin are required but all of the principles pertaining to the use of oxytocin apply to augmentation.

a. Is part of the active management of labor instituted when the labor process is unsatisfactory

The perinatal nurse is caring for Christy following the birth of her first child. Based on Christy's history, the RN recognizes that risk factors for postpartum depression include: Select one or more: a. Loss of friends based on upcoming divorce; family is unable to assist b. Separated from spouse pending divorce c. Unplanned cesarean delivery secondary to Category III fetal tracing d. Good prenatal care with uneventful pregnancy

a. Loss of friends based on upcoming divorce; family is unable to assist b. Separated from spouse pending divorce c. Unplanned cesarean delivery secondary to Category III fetal tracing

Jennifer is 3 hours postpartum following the vaginal delivery of a 9lb 15oz baby girl. Estimated blood loss at delivery was 800 ml. The RN is aware that Jennifer experienced an early-postpartum hemorrhage. Select the appropriate nursing actions for the care of this patient. (Select all that apply.) Select one or more: a. Maintain IV site in case fluids/medication for PPH are indicated b. Frequent fundal assessment to prevent uterine atony and further blood loss c. Assess for displaced uterus secondary to overdistended bladder. d. Assess lochia for amount and for clots

a. Maintain IV site in case fluids/medication for PPH are indicated b. Frequent fundal assessment to prevent uterine atony and further blood loss c. Assess for displaced uterus secondary to overdistended bladder. d. Assess lochia for amount and for clots

A 42-weeks' gestation neonate is admitted to the NICU (neonatal intensive care unit). This neonate is at risk for which complication? Select one: a. Meconium aspiration b. Failure to thrive c. Necrotizing enterocolitis d. Intraventricular hemorrhage

a. Meconium aspiration

Rho immune globulin will be ordered postpartum if which situation occurs? Select one: a. Mother Rh-, baby Rh+ b. Mother Rh+, baby Rh- c. Mother Rh+, baby Rh+ d. Mother Rh-, baby Rh-

a. Mother Rh-, baby Rh+

Which statement by a newly delivered woman indicates that she knows what to expect regarding her menstrual activity after childbirth? a. My first menstrual cycle will be heavier than normal and will return to my prepregnant volume within three or four cycles." b."My first menstrual cycle will be lighter than normal and then will get heavier every month thereafter." c. "I will not have a menstrual cycle for 6 months after childbirth." d. "My first menstrual cycle will be heavier than normal and then will be light for several months after."

a. My first menstrual cycle will be heavier than normal and will return to my prepregnant volume within three or four cycles."

Which diagnostic test is used to confirm a suspected diagnosis of breast cancer? Select one: a. Needle-localization biopsy b. Ultrasound c. Magnetic resonance imaging (MRI) d. Mammogram

a. Needle-localization biopsy

Providing care for the neonate born to a mother who abuses substances can present a challenge for the health care team. Nursing care for this infant requires a multisystem approach. What is the first step in the provision of care for the infant? Select one: a. Neonatal abstinence syndrome (NAS) scoring b. Reduction of environmental stimuli c. Adequate nutrition and maintenance of fluid and electrolyte balance d. Pharmacologic treatment

a. Neonatal abstinence syndrome (NAS) scoring

Allison is 32 weeks pregnant presents to L&D and tells the nurse she has not felt her baby move in 48 hours. The nurse begins electronic fetal monitoring and notes the baby's heart rate is in the 130's. The nurse contacts Allison's physician to obtain an order for which of the following tests? a. Non stress Test b.Fetal fibronectin c.Maternal Serum Screening Test d.Biophysical profile e.Leopolds Maneuvers

a. Non stress Test

Which client(s) should the nurse report to the health care provider? (Select all that apply). Select one or more: a. One 2-3mm sized mobile, nontender mass in lower outer quadrant of left breast b. Left breast slightly smaller than right breast c. Eversion (elevation) of both nipples d. Small dimple located in the upper outer quadrant of the right breast

a. One 2-3mm sized mobile, nontender mass in lower outer quadrant of left breast d. Small dimple located in the upper outer quadrant of the right breast

Tanya, a 30-year-old woman, is being prepared for a planned cesarean birth. The perinatal nurse assists the anesthesiologist with the spinal block and then positions Tanya in a supine position. Tanya's blood pressure drops to 90/52, and there is a decrease in the fetal heart rate to 110 bpm. The perinatal nurse's best response is to: Select one: a. Place Tanya in a left lateral tilt b. Discontinue Tanya's intravenous administration. c. Have naloxone (Narcan) ready for administration. d. Have epinephrine ready for administration.

a. Place Tanya in a left lateral tilt

As part of their teaching function at discharge, nurses should educate parents regarding safe sleep. Based on the most recent evidence, which information is incorrect and should be discussed with parents? Select one: a. Place the infant on his or her abdomen to sleep. b. Avoid loose bedding, water beds, and beanbag chairs. c. Keep the infant away from secondhand smoke. d. Prevent exposure to people with upper respiratory tract infections.

a. Place the infant on his or her abdomen to sleep.

A newborn is jaundiced and is receiving phototherapy via ultraviolet bank lights. What is the most appropriate nursing intervention when caring for an infant with hyperbilirubinemia and receiving phototherapy? Select one: a. Placing eye shields over the newborn's closed eyes b. Applying an oil-based lotion to the newborn's skin to prevent dying and cracking c. Changing the newborn's position every 4 hours d. Limiting the newborn's intake of milk to prevent nausea, vomiting, and diarrhea

a. Placing eye shields over the newborn's closed eyes

A neonate is born at 33 weeks' gestation. This neonate would be classified as: Select one: a. Premature b. Very premature c. Late premature d. Term

a. Premature (Very premature infants are those born at less than 32 weeks' gestation; premature infants are born between 32 and 34 weeks' gestation; late premature are neonates born between 34 and 37 weeks' gestation; term infants are those born between 37-40 weeks' gestation.)

Which of the following are indications for cesarean birth? (Select all that apply.) Select one or more:Read page 788 in textbook and not Box 32-10 a. Previous cesarean birth b. Placental abnormalities c. Previous uterine surgery d. Failure to progress in labor

a. Previous cesarean birth b. Placental abnormalities c. Previous uterine surgery d. Failure to progress in labor

A pregnant woman who has a history of cesarean births is requesting to have a vaginal birth after cesarean (VBAC). In which of the following situations should the nurse advise the patient that her request may be declined? Select one: a. Previous uterine surgery b. Flexed fetal attitude c. Previous low flap uterine incision d. Positive vaginal candidiasis

a. Previous uterine surgery

The perinatal nurse notes a rapid decrease in the fetal heart rate that does not recover immediately following an amniotomy. The most likely cause of this obstetrical emergency is: Select one: a. Prolapsed umbilical cord b. Vasa previa c. Oligohydramnios d. Placental abruption

a. Prolapsed umbilical cord

A primiparous woman is in the taking-in stage of psychosocial recovery and adjustment after childbirth. Recognizing the needs of women during this stage, how should the nurse respond? Select one: a. Provide time for the mother to reflect on the events of her labor and delivery. b. Foster an active role in the baby's care. c. Recognize the woman's limited attention span by giving her written materials to read when she gets home rather than doing a teaching session while she is in the hospital. d. Promote maternal independence by encouraging her to meet her own hygiene and comfort needs.

a. Provide time for the mother to reflect on the events of her labor and delivery.

Which of the following nursing actions are directed at promoting bonding? (Select all that apply.) a. Providing opportunity for parents to hold their newborn as soon as possible following the birth. b. Providing opportunities for the couple to talk about their birth experience and about becoming parents. c.Promoting rest and comfort by keeping the newborn in the nursery at night. d.Providing positive comments to parents regarding their interactions with their newborn.

a. Providing opportunity for parents to hold their newborn as soon as possible following the birth. b. Providing opportunities for the couple to talk about their birth experience and about becoming parents. d.Providing positive comments to parents regarding their interactions with their newborn.

When caring for clients with neoplasms of the reproductive system, the nurse must begin by assessing the woman's knowledge of the disorder, its management, and prognosis. This assessment should be followed by a nursing diagnosis. Which diagnosis fails to address the psychologic effect of these disorders? Select one: a. Risk for injury, related to lack of skill for self-care b. Disturbed body image, as a result of changes in anatomy c. Anxiety, related to surgical procedures d. Interrupted family processes

a. Risk for injury, related to lack of skill for self-care

The induction of labor is considered an acceptable obstetric procedure if it is in the best interest to deliver the fetus. The charge nurse on the labor and delivery unit is often asked to schedule clients for this procedure and therefore must be cognizant of the specific conditions appropriate for labor induction. What are appropriate indications for induction? (Select all that apply) Select one or more: a. Rupture of membranes at or near term b. Convenience of the woman or her physician c. Maternal fatigue and frustration at 38 weeks gestation d. Fetal death e. Postterm pregnancy

a. Rupture of membranes at or near term d. Fetal death e. Postterm pregnancy

Which of the following breath sounds are normal to hear in the neonate during the first few hours postbirth? Select one: a. Scattered crackles b. Wheezes c. Stridor d. Grunting

a. Scattered crackles Scattered crackles may be detected during the first few hours after birth due to retained amniotic fluid. Persistent crackles, wheezes, stridor, grunting, paradoxical breathing, decreased breath sounds, and/or prolonged periods of apnea are signs of respiratory distress. Decreased or absent breath sounds are often related to meconium aspiration or pneumothorax.

A pregnant woman at 37 weeks of gestation has had ruptured membranes for 26 hours. A cesarean section is performed for failure to progress. The fetal heart rate (FHR) before birth is 180 beats per minute with limited variability. At birth the newborn has Apgar scores of 6 and 7 at 1 and 5 minutes and is noted to be pale and tachypneic. Based on the maternal history, what is the most likely cause of this newborn's distress? Select one: a. Sepsis b. Respiratory distress syndrome c. Phrenic nerve injury d. Hypoglycemia

a. Sepsis

A health care provider performs a clinical breast examination on a woman diagnosed with fibroadenoma. How would the nurse explain the defining characteristics of a fibroadenoma? (Select all that apply). Select one or more: a. Single lump in one breast that can increase in size. b. Thick, sticky discharge from the nipple of the affected breast c. Does not increase risk for breast cancer d. Lumpiness in both breasts that develops 1 week before menstruation e. Inflammation of the milk ducts and glands behind the nipples

a. Single lump in one breast that can increase in size. c. Does not increase risk for breast cancer

A pregnant woman at term is transported to the emergency department (ED) after a severe vehicular accident. The obstetric nurse responds and rushes to the ED with a fetal monitor. Cardiopulmonary arrest occurs as the obstetric nurse arrives. What is the highest priority for the trauma team? Select one: a. Starting cardiopulmonary resuscitation (CPR) b. Obtaining IV access, and starting aggressive fluid resuscitation c. Quickly applying the fetal monitor to determine whether the fetus viability d. Transferring the woman to the surgical unit for an emergency cesarean delivery in case the fetus is still alive

a. Starting cardiopulmonary resuscitation (CPR)

Pain should be regularly assessed in all newborns. If the infant is displaying physiologic or behavioral cues that indicate pain, then measures should be taken to manage the pain. Which interventions are examples of nonpharmacologic pain management techniques? (Select all that apply.) Select one or more: a. Sucrose b. Skin-to-skin contact with the mother c. Acetaminophen d. Swaddling e. Nonnutritive sucking

a. Sucrose b. Skin-to-skin contact with the mother d. Swaddling e. Nonnutritive sucking

One of the most important components of the physical assessment of the pregnant client is the determination of BP. Consistency in measurement techniques must be maintained to ensure that the nuances in the variations of the BP readings are not the result of provider error. Which techniques are important in obtaining accurate BP readings? (Select all that apply.) Select one or more: a. The client should be seated b. The client's arm should be placed at the level of the heart. c. An electronic BP device should be used. d. The cuff should cover a minimum of 60% of the upper arm. e. The same arm should be used for every reading

a. The client should be seated b. The client's arm should be placed at the level of the heart. e. The same arm should be used for every reading

A nurse working with an infertile couple has made the following nursing diagnosis: Sexual dysfunction related to decreased libido. Which of the following assessments is the likely reason for this diagnosis? Select one: a. The couple has established a set schedule for their sexual encounters. b. The couple has been married for more than 8 years. c. The couple lives with one set of parents. d. The couple has close friends who gave birth within the last year.

a. The couple has established a set schedule for their sexual encounters.

A nurse is providing education to a support group of women newly diagnosed with breast cancer. It is important for the nurse to discuss which factor related to breast cancer with the group? Select one: a. The exact cause of breast cancer remains unknown. b. In the United States, 1 in 10 women will develop breast cancer in her lifetime. c. Genetic mutations account for 50% of women who will develop breast cancer. d. Breast cancer is the leading cause of cancer death in women.

a. The exact cause of breast cancer remains unknown.

A woman at 24 weeks of gestation states that she has a glass of wine with dinner every evening. Why would the nurse counsel the client to eliminate all alcohol? Select one: a. The fetus is placed at risk for altered brain growth. b. Daily consumption of alcohol indicates a risk for alcoholism. c. She will be at risk for abusing other substances as well. d. The fetus is at risk for multiple organ anomalies.

a. The fetus is placed at risk for altered brain growth.

A couple is undergoing an infertility workup. The semen analysis indicates a decreased number of sperm and immature sperm. Which of the following factors can have a potential effect on sperm maturity? (Select all that apply.) Select one or more: a. The man rides a bike to and from work each day. b. The man takes a calcium channel blocker for the treatment of hypertension. c. The man drinks 6 cups of coffee a day. d. The man was treated for prostatitis 12 months ago and has been symptom free since treatment.

a. The man rides a bike to and from work each day. b. The man takes a calcium channel blocker for the treatment of hypertension.

The postpartum nurse caring for a 20-year-old G1 P1 woman who 3 hours ago delivered a healthy full-term infant, observes the woman who is lightly touching her baby girl with her fingertips but who seems to be uncomfortable holding her baby close to her body. Which of the following is an accurate interpretation of these observed behaviors? Select one: a. The woman is demonstrating behavior that facilitates infant attachment. b. The woman is in the taking-in phase. c. The woman is having difficulty in bonding with her baby. d. The woman needs to be medicated for pain.

a. The woman is demonstrating behavior that facilitates infant attachment.

27 year old Amy G0 presents to the clinic for a preconceptual visit. Her BMI is 41. What would the nurse include in their education for Amy about pregnancy and a high BMI? (Select all that apply). Select one or more: a. There is increased risk for preterm birth. b. There is decreased risk for miscarriage. c. There is increased risk for blood clot in her legs and lungs. d. There is decreased risk for cesarean section. e. There is increased risk for gestational diabetes.

a. There is increased risk for preterm birth. c. There is increased risk for blood clot in her legs and lungs. e. There is increased risk for gestational diabetes.

A woman who has recently given birth complains of pain and tenderness in her leg. On physical examination, the nurse notices warmth and redness over an enlarged, hardened area. Which condition should the nurse suspect, and how will it be confirmed? Select one: a. Thrombophlebitis; using real-time and color Doppler ultrasound b. Idiopathic or immune thrombocytopenic purpura (ITP); drawing blood for laboratory analysis c. Disseminated intravascular coagulation (DIC); asking for laboratory tests d. von Willebrand disease (vWD); noting whether bleeding times have been extended

a. Thrombophlebitis; using real-time and color Doppler ultrasound

A woman in preterm labor at 30 weeks of gestation receives two 12-mg intramuscular (IM) doses of betamethasone. What is the purpose of this pharmacologic intervention? Select one: a. To stimulate fetal surfactant production Correct! Important as baby may deliver preterm. b. To reduce maternal and fetal tachycardia associated with ritodrine administration c. To maintain adequate maternal respiratory effort and ventilation during magnesium sulfate therapy d. To suppress uterine contractions

a. To stimulate fetal surfactant production Correct! Important as baby may deliver preterm.

During the postpartum assessment, the perinatal nurse notes that a patient who has just experienced a forceps-assisted birth now has a large quantity of bright red bleeding. Her uterine fundus is firm and midline. The nurse's most appropriate action is to notify the physician/certified nurse midwife and anticipate the need for: Select one: a. Vaginal assessment and possible repair of vaginal and/or cervical lacerations b. An oxytocin infusion c. Further information for the woman/family about forceps d. Bladder assessment and catheterization

a. Vaginal assessment and possible repair of vaginal and/or cervical lacerations

Approximately 10% to 15% of all clinically recognized pregnancies end in miscarriage. What are possible causes of early miscarriage? (Select all that apply.) Select one or more: a. Varicella b. Endocrine imbalance c. Chromosomal abnormalities d. Nausea and vomiting in early pregnancy e. Systemic disorders

a. Varicella b. Endocrine imbalance c. Chromosomal abnormalities e. Systemic disorders

Nursing actions focused at reducing a postpartum woman's risk for cystitis include which of the following? (Select all that apply.) Select one or more: a. Voiding within a few hours post-birth b. Oral intake of a minimum of 3000 mL per day c. Changing peri-pads every 3 to 4 hours or more frequently as indicated d. Reminding the woman to void every 3 to 4 hours while awake

a. Voiding within a few hours post-birth b. Oral intake of a minimum of 3000 mL per day c. Changing peri-pads every 3 to 4 hours or more frequently as indicated d. Reminding the woman to void every 3 to 4 hours while awake

The nurse is teaching the parents of a healthy newborn about infant safety. Which of the following should be included in the teaching plan? (Select all that apply). Select one or more: a. Water temperature for the infant's bath should be 100.4 degrees F. b. Do not cook while holding an infant c. Cover electrical outlets d. Remove strings from infant sleepwear, bedding, and pacifiers to prevent strangulation.

a. Water temperature for the infant's bath should be 100.4 degrees F. b. Do not cook while holding an infant c. Cover electrical outlets d. Remove strings from infant sleepwear, bedding, and pacifiers to prevent strangulation.

A nurse notes that an Eskimo woman does not cuddle or interact with her newborn other than to feed him, change his diapers or soiled clothes, and put him to bed. While evaluating this client's behavior with her infant, what realization does the nurse make? Select one: a. What appears to be a lack of interest in the newborn is, in fact, the cultural way of demonstrating intense love by attempting to ward off evil spirits. b. The woman needs a referral to a social worker for further evaluation of her parenting behaviors once she goes home with the newborn. c. Extra time needs to be planned for assisting the woman in bonding with her newborn. d. The woman is inexperienced in caring for a newborn.

a. What appears to be a lack of interest in the newborn is, in fact, the cultural way of demonstrating intense love by attempting to ward off evil spirits.

What information should the nurse understand fully regarding rubella and Rh status? Select one: a. Women should be warned that the rubella vaccination is teratogenic and that they must avoid pregnancy for at least 1 month after vaccination b. Rh immunoglobulin boosts the immune system and thereby enhances the effectiveness of vaccinations. c. Breastfeeding mothers cannot be vaccinated with the live attenuated rubella virus. d. Rh immunoglobulin is safely administered intravenously because it cannot harm a nursing infant.

a. Women should be warned that the rubella vaccination is teratogenic and that they must avoid pregnancy for at least 1 month after vaccination

Which information regarding the care of antepartum women with cardiac conditions is most important for the nurse to understand? Select one: a. Women with class II cardiac disease should avoid heavy exertion and any activity that causes even minor symptoms. b. Stress on the heart is greatest in the first trimester and the last 2 weeks before labor. c. Women with class III cardiac disease should get 8 to 10 hours of sleep every day and limit housework, shopping, and exercise. d. Women with class I cardiac disease need bed rest through most of the pregnancy and face the possibility of hospitalization near term.

a. Women with class II cardiac disease should avoid heavy exertion and any activity that causes even minor symptoms.

According to the CDC,which of the following are significant causes of maternal death in the United States? (Select all that apply). Select one or more: a. cardio-vascular disease b. street drug use c. ski accidents d. hypertensive disorders e. non-cardiovascular conditions f. hemorrhage

a. cardio-vascular disease d. hypertensive disorders e. non-cardiovascular conditions f. hemorrhage

Amy is a 20 year old sexually active woman (G0000) who wants an effective birth control method. She denies any medical problems. Which of the following would the nurse not recommend to Amy? Select one: a. essure b. intrauterine device c. the implant d. oral contraceptive pills

a. essure

Ultrasound can be used in antepartum care for which of the following assessments? (Select all that apply). Select one or more: a. fetal growth b. gestational age c. placental position and function d. adjunct use in chorionic villi sampling (CVS)

a. fetal growth b. gestational age c. placental position and function d. adjunct use in chorionic villi sampling (CVS)

Pre-surgical nursing care for the woman who is going to have a hysterectomy for ovarian cancer includes which of the following? (Select all that apply). Select one or more: a. identification band is in place b. Sips of water for hydration c. ensure lab results are available to the surgeon prior to surgery d. administer enema if ordered e. teach post-op routine care such as need for early ambulation

a. identification band is in place c. ensure lab results are available to the surgeon prior to surgery d. administer enema if ordered e. teach post-op routine care such as need for early ambulation

Which of the following is true regarding infertility? (Select all that apply). Select one or more: a. infertility is defined as the inability to achieve pregnancy after a year of unprotected intercourse b. approx 60% of infertility problems are related to the female factors c. anovulation can cause primary and/or secondary infertity d. infertility effects approx 5% of reproductive-age couples

a. infertility is defined as the inability to achieve pregnancy after a year of unprotected intercourse c. anovulation can cause primary and/or secondary infertity

The CHOICE Project removed 3 key barriers to contraception for many women. These included which of the following? Select one or more: a. lack of access to preferred method b. state laws that restrict methods c. cost d. parental permission e. knowledge deficit

a. lack of access to preferred method c. cost e. knowledge deficit

Yolanda is 6 weeks pregnant by dates and is considering abortion. What options might be appropriate for her at this point? (Select all that apply). Select one or more: a. medical abortion with mifepristone and misopristol b. surgical abortion with aspiration see c. emergency contraception d. it is too late for her to have an abortion

a. medical abortion with mifepristone and misopristol b. surgical abortion with aspiration see

An infant at 26 weeks of gestation arrives intubated from the delivery room. The nurse weighs the infant, places him under the radiant warmer, and attaches him to the ventilator at the prescribed settings. A pulse oximeter and cardiorespiratory monitor are placed. The pulse oximeter is recording oxygen saturations of 80%. The prescribed saturations are 92%. What are the nurse's most appropriate actions at this time? Select one or more: a. notify physician b. listen to breath sounds c. ensure patency of the endotracheal tube d. notify parents that their infant is not doing well e. increase oxygen f. complete the admission process and thorough assessment g. continue to observe until saturations are 75%

a. notify physician b. listen to breath sounds c. ensure patency of the endotracheal tube e. increase oxygen

Which of the following are management options for couples faced with infertility challenges, depending of the cause? (Select all that apply). Select one or more: a. remain child-free see b. assisted reproductive technology procedures c. Medication d. adoption

a. remain child-free see b. assisted reproductive technology procedures c. Medication d. adoption

16 year old Emily lives in Blackfoot, ID. She has a boyfriend but does not want to have a baby for at least 10 years. She wants an effective, easy birth control method and denies medical problems. She has periods every 4-6 weeks. The nurse would recommend which of the following methods to Emily? (Select all that apply). Select one or more: a. the implant b. natural family planning c. bilateral tubal ligation d. hormonal IUD

a. the implant d. hormonal IUd

A nurse is discussing the storage of breast milk with a mother whose infant is preterm and in the special care nursery. Which statement indicates that the mother requires additional teaching? a."I can store my breast milk in the refrigerator for 3 months." b."I can store my breast milk in the refrigerator for 3 to 5 days." c."I can store my breast milk at room temperature for 4 hours." d."I can store my breast milk in the freezer for 3 months."

a."I can store my breast milk in the refrigerator for 3 months."

Tachysystole, previously referred to as hyperstimulation, is defined as: a.Contractions lasting 2 minutes or longer b.Five or more contractions in 10 minutes over a 30-minute window c.Contractions occurring within 1 minute of each other d.Uterine resting tone below 20mm/Hg

a.Contractions lasting 2 minutes or longer b.Five or more contractions in 10 minutes over a 30-minute window c.Contractions occurring within 1 minute of each other

During a prenatal visit, the nurse is explaining dietary management to a woman with pregestational diabetes. Which statement by the client reassures the nurse that teaching has been effective? a.Diet and insulin needs change during pregnancy b.I will need to eat 600 more calories per day because I am pregnant. c.I will plan my diet based on the results of urine glucose testing. d.I can continue with the same diet as before pregnancy as long as it is well balanced.

a.Diet and insulin needs change during pregnancy

A pregnant woman was admitted for induction of labor at 43 weeks of gestation with sure dates. A nonstress test (NST) in the obstetrician's office revealed a nonreactive tracing. On artificial rupture of membranes, thick meconium-stained fluid was noted. What should the nurse caring for the infant after birth anticipate? (Select all that apply). a.Meconium aspiration b.Hypoglycemia c.excessive vernix caseosa covering the skin d.Increased amount of subcutaneous fat e.Dry, cracked skin f.Absence of scalp hair

a.Meconium aspiration b.Hypoglycemia e.Dry, cracked skin

For which of the following conditions is Daily Fetal Movement Count indicated? (Select all that apply). Select one or more: a.Mother with diabetes during pregnancy b.Mother who complains of headaches during pregnancy that are relieved with Tylenol c. Mother with low amniotic fluid volume or oligohydraminos d.Pregnant woman who is older than 35 years with no complications

a.Mother with diabetes during pregnancy c. Mother with low amniotic fluid volume or oligohydraminos

What are the most common causes for subinvolution of the uterus? a.Retained placental fragments and infection b.Uterine tetany and overproduction of oxytocin c.Multiple gestation and postpartum hemorrhage d.Postpartum hemorrhage and infection

a.Retained placental fragments and infection

Which statement regarding hemolytic diseases of the newborn is most accurate? a.The indirect Coombs' test is performed on the mother before birth; the direct Coombs' test is performed on the cord blood after birth. b.Rh incompatibility matters only when an Rh-negative child is born to an Rh-positive mother. c.ABO incompatibility is more likely than Rh incompatibility to precipitate significant anemia. d.Exchange transfusions are frequently required in the treatment of hemolytic disorders.

a.The indirect Coombs' test is performed on the mother before birth; the direct Coombs' test is performed on the cord blood after birth.

A mother is changing the diaper of her newborn son and notices that his scrotum appears large and swollen. The client is concerned. What is the best response from the nurse? Select one: a. "Your baby's urine is backing up into his scrotum." b. "A large scrotum and swelling indicate a hydrocele, which is a common finding in male newborns." c. "I don't know, but I'm sure it is nothing." d. "Your baby might have testicular cancer."

b. "A large scrotum and swelling indicate a hydrocele, which is a common finding in male newborns."

Your pregnant patient is in her first trimester and is scheduled for an ultrasound. When explaining the rationale for early pregnancy ultrasound, the best response is: Select one: a. "The test will help to determine the baby's position." b. "The test will help to determine how many weeks you are pregnant." c. "The test will help to determine if your baby is growing appropriately." d. "The test will help to determine if you have a boy or girl."

b. "The test will help to determine how many weeks you are pregnant."

Which options for saying "good-bye" would the nurse want to discuss with a woman who is diagnosed with having a stillborn girl? Select one: a. "What funeral home do you want notified after the baby is born?" b. "When your baby is born, would you like to see and hold her?" c. The nurse should not discuss any options at this time; plenty of time will be available after the baby is born. d. "Would you like a picture taken of your baby after birth?"

b. "When your baby is born, would you like to see and hold her?"

On day 3 of life, a newborn continues to require 100% oxygen by nasal cannula. The parents ask if they may hold their infant during his next gavage feeding. Considering that this newborn is physiologically stable, what response should the nurse provide? Select one: a. "Parents are not allowed to hold their infants who are dependent on oxygen." b. "You may hold your baby during the feeding." c. "You may only hold your baby's hand during the feeding." d. "Feedings cause more physiologic stress; therefore, the baby must be closely monitored. I don't think you should hold the baby."

b. "You may hold your baby during the feeding."

A woman gave birth to an infant boy 10 hours ago. Where does the nurse expect to locate this woman's fundus? Select one: a. Nonpalpable abdominally b. 1 centimeter above the umbilicus c. 2 centimeters below the umbilicus d. Midway between the umbilicus and the symphysis pubis

b. 1 centimeter above the umbilicus

Sandy G21001 is 41 weeks pregnant and presents to L&D for induction. The physician orders Pitocin to be infused at 2mU/min. The IV bag is LR 1000mL with 10 units of Pitocin. The infusion pump delivers ___mL/hour. How will the nurse program the infusion pump to deliver the medication as ordered? Select one: a. 18mL/hour b. 12 mL/hour c. 30mL/hour d. 6mL/hour

b. 12 mL/hour 10 units:1000mL:: X:1mL 1000X=10 X=0.01unit or 10 mU 2mU/min divided by 10mU/mL = 0.2mL/min x 60 min = 12 mL/hour

An infant at 36 weeks of gestation has increasing respirations (80 to 100 breaths per minute with significant substernal retractions). The infant is given oxygen by continuous nasal positive airway pressure (CPAP). What level of partial pressure of arterial oxygen (PaO2) indicates hypoxia? Select one: a. 67 mmHg b. 45 mmHg c. 89 mmHg d. 73 mmHg

b. 45 mmHg

At 1 minute after birth a nurse assesses an infant and notes a heart rate of 80 beats per minute, some flexion of extremities, a weak cry, grimacing, and a pink body but blue extremities. Which Apgar score does the nurse calculate based upon these observations and signs? Select one: a. 4 b. 5 c. 7 d. 6

b. 5

A 25-year-old gravida 1 para 1 who had an emergency cesarean birth 3 days ago is scheduled for discharge. As the nurse prepares her for discharge, she begins to cry. The nurse's next action should be what? Select one: a. Assess her for pain. b. Allow her time to express her feelings. c. Point out how lucky she is to have a healthy baby. d. Explain that she is experiencing postpartum blues

b. Allow her time to express her feelings.

A 25 year-old woman gave birth to her second child 6 hours ago. She informs the nurse that she is bleeding more than with her previous birth experience. The initial nursing action is to: Select one: a. Explain that this is normal for second-time moms. b. Assess the location and firmness of the fundus c. Change her pad and return in 1 hour and reassess. d. Give her 10 units of oxytocin as per standing order.

b. Assess the location and firmness of the fundus

The nurse is performing an assessment on a client who thinks she may be experiencing preterm labor. Which information is the most important for the nurse to understand and share with the client? Select one: a. Because preterm labor is likely to be the start of an extended labor, a woman with symptoms can wait several hours before contacting the primary caregiver. b. Diagnosis of preterm labor is based on gestational age, uterine activity, and progressive cervical change. c. Braxton Hicks contractions often signal the onset of preterm labor. d. Because all women must be considered at risk for preterm labor and prediction is so variable, teaching pregnant women the symptoms of preterm labor probably causes more harm through false alarms.

b. Diagnosis of preterm labor is based on gestational age, uterine activity, and progressive cervical change.

Which is the most accurate description of PPD without psychotic features? Select one: a. Condition that disappears without outside help b. Distinguishable by irritability c. Postpartum baby blues requiring the woman to visit with a counselor or psychologist d. Condition that is more common among older Caucasian women because they have higher expectations

b. Distinguishable by irritability

Breast pain occurs in many women during their perimenopausal years. Which information is (are) a priority for the nurse to share with the client? (Select all that apply). Select one or more: a. Pain is almost always an indication of a solid mass. b. Distinguishing between cyclical and noncyclical pain is important. c. Assess for prior trauma or possible infection d. Breast pain is an early indication of cancer.

b. Distinguishing between cyclical and noncyclical pain is important. c. Assess for prior trauma or possible infection

A number of metabolic changes occur throughout pregnancy. Which physiologic adaptation of pregnancy will influence the nurse's plan of care? Select one: a. Insulin crosses the placenta to the fetus only in the first trimester, after which the fetus secretes its own. b. During the second and third trimesters, pregnancy exerts a diabetogenic effect that ensures an abundant supply of glucose for the fetus c. Maternal insulin requirements steadily decline during pregnancy. d. Women with insulin-dependent diabetes are prone to hyperglycemia during the first trimester because they are consuming more sugar.

b. During the second and third trimesters, pregnancy exerts a diabetogenic effect that ensures an abundant supply of glucose for the fetus

Which finding on a prenatal visit at 10 weeks of gestation might suggest a hydatidiform mole? (Select all that apply). Select one or more: a. History of bright red spotting for 1 day, weeks ago b. Fundal height measurement of 18 cm c. Blood pressure of 120/80 mm Hg d. Complaint of severe nausea

b. Fundal height measurement of 18 cm d. Complaint of severe nausea

Which of the following antepartum tests is (are) used to evaluate a Maternal Serum Screening test that is negative for increased risk? Select one: a. Biophysical profile b. Further followup test not necessary c. CVS d. Amniocentesis

b. Further followup test not necessary

A nurse is providing breast care education to a client after mammography. Which information regarding fibrocystic changes in the breast is important for the nurse to share? Select one: a. Fibrocystic breast disease is a disease of the milk ducts and glands in the breasts b. Healthy women with fibrocystic breast disease find lumpiness with pain and tenderness in varying degrees in the breast tissue during menstrual cycles. c. Lumpiness is accompanied by tenderness after menses. d. It is a premalignant disorder characterized by lumps found in the breast tissue.

b. Healthy women with fibrocystic breast disease find lumpiness with pain and tenderness in varying degrees in the breast tissue during menstrual cycles.

A woman with severe preeclampsia has been receiving magnesium sulfate by intravenous infusion for 8 hours. The nurse assesses the client and documents the following findings: temperature of 37.1° C, pulse rate of 96 beats per minute, respiratory rate of 24 breaths per minute, BP of 155/112 mm Hg, 3+ DTRs, and no ankle clonus. The nurse calls the provider with an update. The nurse should anticipate an order for which medication? Select one: a. Magnesium sulfate bolus b. Hydralazine c. Calcium gluconate d. Diazepam

b. Hydralazine

A woman with severe preeclampsia has been receiving magnesium sulfate by intravenous infusion for 8 hours. The nurse assesses the client and documents the following findings: temperature of 37.1° C, pulse rate of 96 beats per minute, respiratory rate of 24 breaths per minute, BP of 155/112 mm Hg, 3+ DTRs, and no ankle clonus. The nurse calls the provider with an update. The nurse should anticipate an order for which medication? Select one: a. Magnesium sulfate bolus b. Hydralazine c. Diazepam d. Calcium gluconate

b. Hydralazine

A 26-year-old primigravida has come to the clinic for her regular prenatal visit at 12 weeks. She appears thin and somewhat nervous. She reports that she eats a well-balanced diet, although her weight is 5 pounds less than it was at her last visit. The results of laboratory studies confirm that she has a hyperthyroid condition. Based on the available data, the nurse formulates a plan of care. Which nursing diagnosis is most appropriate for the client at this time? Select one: a. Deficient fluid volume b. Imbalanced nutrition: less than body requirements c. Imbalanced nutrition: more than body requirements d. Disturbed sleep pattern

b. Imbalanced nutrition: less than body requirements

Which of the following neonatal signs or symptoms would the nurse expect to see in a breastfed neonate with an elevated bilirubin level in the first 24 hours of life? Select one: a. Low glucose b. Ineffective breastfeeding c. Hyperactivity d. Hyperthermia

b. Ineffective breastfeeding

Which concerns regarding parenthood are often expressed by visually impaired mothers? (Select all that apply.) Select one or more: a. Ability to care for the infant b. Infant safety c. Needing extra time for parenting activities to accommodate the visual limitations d. Transportation

b. Infant safety c. Needing extra time for parenting activities to accommodate the visual limitations d. Transportation

What is the most critical physiologic change required of the newborn after birth? Select one: a. Maintenance of a stable temperature b. Initiation and maintenance of respirations c. Full function of the immune defense system d. Closure of fetal shunts in the circulatory system

b. Initiation and maintenance of respirations

Reports have linked third trimester use of selective serotonin uptake inhibitors (SSRIs) with a constellation of neonatal signs. The nurse is about to perform an assessment on the infant of a mother with a history of a mood disorder. Which signs and symptoms in the neonate may be the result of maternal SSRI use? (Select all that apply.) Select one or more: a. Hyperglycemia b. Irritability c. Shivering d. Fever e. Hypotonia

b. Irritability c. Shivering d. Fever

Screening at 24 weeks of gestation reveals that a pregnant woman has gestational diabetes mellitus (GDM). In planning her care, the nurse and the client mutually agree that an expected outcome is to prevent injury to the fetus as a result of GDM. This fetus is at the greatest risk for which condition? Select one: a. Preterm birth b. Macrosomia c. Low birth weight d. Congenital anomalies of the central nervous system

b. Macrosomia

Karen, a G2 P2, experienced a precipitous birth 90 minutes ago. Her infant is 4200 grams and a repair of a second-degree laceration was needed following the birth. As part of the nursing assessment, the nurse discovers that Karen's uterus is boggy. Furthermore, it is noted that Karen's vaginal bleeding has increased. What is the nurse's most appropriate actions? Select one or more: a. Assess vital signs including blood pressure and pulse. b. Massage the uterine fundus with continual lower segment support. c. Measure and document each perineal pad changed in order to assess blood loss. d. Assess for bladder distention and encourage patient to void.

b. Massage the uterine fundus with continual lower segment support. d. Assess for bladder distention and encourage patient to void.

The early postpartum period is a time of emotional and physical vulnerability. Many mothers can easily become psychologically overwhelmed by the reality of their new parental responsibilities. Fatigue compounds these issues. Although the baby blues are a common occurrence in the postpartum period, approximately 500,000 women in America experience a more severe syndrome known as PPD. Which statement regarding PPD is essential for the nurse to be aware of when attempting to formulate a nursing diagnosis? Select one: a. This syndrome affects only new mothers. b. PPD can easily go undetected. c. PPD symptoms are consistently severe. d. Only mental health professionals should teach new parents about this condition.

b. PPD can easily go undetected.

The nurse assesses that a full-term neonate's temperature is 97.1°F (36.2°C). The first nursing action is to: Select one: a. Turn up the heat in the room. b. Place the neonate on the mother's chest with a warm blanket over the mother and baby. c. Take the neonate to the nursery and place in a radiant warmer. d. Notify the neonate's primary provider.

b. Place the neonate on the mother's chest with a warm blanket over the mother and baby. Preventive nursing actions to prevent cold stress include skin-to-skin contact with the mother with a warm blanket over both.

Which adverse prenatal outcomes are associated with the HELLP syndrome? (Select all that apply.) Select one or more: a. Placenta previa b. Placental abruption c. Maternal and fetal death d. Cirrhosis e. Renal failure

b. Placental abruption c. Maternal and fetal death e. Renal failure

Typical signs of abusive head trauma (AHT, also known as Shaken Baby Syndrome) include which of the following? (Select all that apply.) Select one or more: a. Broken clavicle b. Poor feeding c. Vomiting d. Breathing problems

b. Poor feeding c. Vomiting d. Breathing problems

If a woman is at risk for thrombus and is not ready to ambulate, which nursing intervention would the nurse use? (Select all that apply.) Select one or more: a. Promoting bed rest b. Putting her in antiembolic stockings (thromboembolic deterrent [TED] hose) and/or sequential compression device (SCD) boots c. Having her flex, extend, and rotate her feet, ankles, and legs d. Immediately notifying the physician if a positive Homans sign occurs e. Having her sit in a chair

b. Putting her in antiembolic stockings (thromboembolic deterrent [TED] hose) and/or sequential compression device (SCD) boots c. Having her flex, extend, and rotate her feet, ankles, and legs d. Immediately notifying the physician if a positive Homans sign occurs

What should the nurse's next action be if the client's white blood cell (WBC) count is 25,000/mm3 on her second postpartum day? Select one: a. Immediately begin antibiotic therapy. b. Recognize that this count is an acceptable range at this point postpartum c. Have the laboratory draw blood for reanalysis. d. Immediately inform the physician.

b. Recognize that this count is an acceptable range at this point postpartum

A primigravida woman at 42 weeks' gestation received Prepidil (dinoprostone) for induction 12 hours ago. The Bishop score is now 3. Which of the following actions by the nurse is appropriate? Select one: a. Perform Nitrazine analysis of the amniotic fluid. b. Report the lack of progress to the obstetrician. c. Place the woman on her left side. d. Ask the provider for an order for oxytocin.

b. Report the lack of progress to the obstetrician. Prepidil is indicated for cervical ripening, the process of physical softening and opening of the cervix. Cervical status is the most important predictor of successful induction of labor. Cervical status is assessed before induction of labor using the Bishop score. A score of 6 or more is considered favorable for successful induction of labor.

Which of the following nursing actions are directed at assisting men in their transition to fatherhood? (Select all that apply.) Select one or more: a. Encourage the woman to take on the major responsibility for infant care. b. Talk to the couple about their expectations of the parenting role. c. Praise the father for his interactions with his infant. d. Provide information on infant care and behavior to both parents

b. Talk to the couple about their expectations of the parenting role. c. Praise the father for his interactions with his infant. d. Provide information on infant care and behavior to both parents

A woman with preeclampsia has a seizure. What is the nurse's highest priority during a seizure? Select one: a. To administer oxygen by mask b. To stay with the client and call for help c. To insert an oral airway d. To suction the mouth to prevent aspiration

b. To stay with the client and call for help

A client at 39 weeks of gestation has been admitted for an external version. Which intervention would the nurse anticipate the provider to order? Select one: a. Contraction stress test (CST) b. Tocolytic drug c. Local anesthetic d. Foley catheter

b. Tocolytic drug

A 26-year-old pregnant woman, gravida 2, para 1-0-0-1, is 28 weeks pregnant when she experiences bright red, painless vaginal bleeding. On her arrival at the hospital, which diagnostic procedure will the client most likely have performed? Select one: a. Internal fetal monitoring b. Transvaginal ultrasound for placental location c. Amniocentesis for fetal lung maturity d. Contraction stress test (CST)

b. Transvaginal ultrasound for placental location

Cellulitis with or without abscess formation is a fairly common condition. The nurse is providing education for a client whose presentation to the emergency department includes an infection of the breast. Which information should the nurse share with this client? (Select all that apply.) Select one or more: a. Breast is pale in color and cool to the touch. b. Treatment for cellulitis or breast infection will include antibiotics. c. Methicillin- resistant Streptococcus aureus is the most common pathogen. d. Nipple piercing may be the cause of a recent infection e. Obesity, smoking, and diabetes are risk factors.

b. Treatment for cellulitis or breast infection will include antibiotics. d. Nipple piercing may be the cause of a recent infection e. Obesity, smoking, and diabetes are risk factors.

The perinatal nurse demonstrates for the student nurse the correct technique of postpartum uterine palpation. Support for the lower uterine segment is critical, as without it, there is an increased risk of: Select one: a. Uterine edema b. Uterine inversion c. Incorrect measurement d. Intensifying the patient's level of pain

b. Uterine inversion

The perinatal nurse understands that the purpose of the surgical "time-out" is to: Select one: a. Confirm that the surgeon is ready to begin b. Verify that it is the correct patient and planned procedure c. Verify that anesthesia is adequate d. Confirm that the neonatal team is in attendance

b. Verify that it is the correct patient and planned procedure

New screening mammogram recommendations by ACOG include which of the following? Select one: a. All women after 55 y/o should have annual screening mammograms b. Women should talk to their health care provider to decide the best age to begin screening mammograms c. All women begin screening mammogram at 40 years old d. Only women with increased risk for breast begin screening mammogram at age 40

b. Women should talk to their health care provider to decide the best age to begin screening mammograms

For which of the following conditions is colposcopy used to further evaluate need for intervention? (Select all that apply). Select one or more: a. bacterial vaginosis b. vaginal cancer c. uterine fibroids d. uterine cancer e. cervical cancer

b. vaginal cancer e. cervical cancer

A baby has just been admitted into the neonatal intensive care unit with a diagnosis of intrauterine growth restriction (IUGR). Which of the following maternal problems could have resulted in this complication? (Select all that apply.) a.Cholecystitis b.Chronic Hypertension with Preeclampsia c.Cigarette Smoker d. Candidiasis e.Cerebral Palsy

b.Chronic Hypertension with Preeclampsia c.Cigarette Smoker

The postpartum mother asks the nurse why is it so important to prevent cold stress in her baby- can't she shiver to stay warm? What should the nurse include in their response? (Select all that apply) a.Yes, your baby can shiver; but it important that we prevent her from needing to so. b.If your baby gets too cold and we don't help her, her body will use alot of oxygen to try and get warm, which can ultimately lead to decrease oxygen to her lungs and body causing respiratory problems. c.Your baby will eventually warm up on her own, but it is best if we keep the hat on her head. d.No, your baby cannot shiver; therefore, it is important to prevent her from losing body heat.

b.If your baby gets too cold and we don't help her, her body will use alot of oxygen to try and get warm, which can ultimately lead to decrease oxygen to her lungs and body causing respiratory problems. d.No, your baby cannot shiver; therefore, it is important to prevent her from losing body heat.

23 y/o Amy delivered a baby 2 months ago and presents to the clinic with complaint of stool coming out of her vagina for the past week. The nurse suspects recto-vaginal fistula. What is the next most appropriate question the nurse should ask Amy? Select one: a. "How long has this been going on?" b. "Are you sure it is stool?" c. "Did you deliver your baby vaginally?" d. "How big was your baby?

c. "Did you deliver your baby vaginally?"

A new mother wants to be sure that she is meeting her daughter's needs while feeding the baby commercially prepared infant formula. The nurse should evaluate the mother's knowledge about appropriate infant feeding techniques. Which statement by the client reassures the nurse that correct learning has taken place? Select one: a. "I refrigerate any leftover formula for the next feeding." b. "I warm the bottle in my microwave oven." c. "I burp my daughter during and after the feeding as needed." d. "Since reaching 2 weeks of age, I add rice cereal to my daughter's formula to ensure adequate nutrition.

c. "I burp my daughter during and after the feeding as needed."

Pelvic floor exercises, also known as Kegel exercises, will help to strengthen the perineal muscles and encourage healing after childbirth. The nurse requests the client to repeat back instructions for this exercise. Which response by the client indicates successful learning? Select one: a. "I contract my thighs, buttocks, and abdomen." b. "I perform 10 of these exercises every day." c. "I pretend that I am trying to stop the flow of urine in midstream." d. "I stand while practicing this new exercise routine."

c. "I pretend that I am trying to stop the flow of urine in midstream."

A woman with worsening preeclampsia is admitted to the hospital's labor and birth unit. The physician explains the plan of care for severe preeclampsia, including the induction of labor, to the woman and her husband. Which statement by the husband leads the nurse to believe that the couple needs further information? Select one: a. "I will stay with my wife during her labor, just as we planned." b. "I will help my wife use the breathing techniques that we learned in our childbirth classes." c. "Since we will be here for a while, I will call my mother so she can bring the two boys—2 years and 4 years of age—to visit their mother." d. "I will give my wife ice chips to eat during labor."

c. "Since we will be here for a while, I will call my mother so she can bring the two boys—2 years and 4 years of age—to visit their mother."

While taking a family history, 37 y/o Bernice tells the nurse her paternal grandmother had breast and colon cancer in her 60's, two paternal aunts had breast cancer in their 40's, one of those aunts also had ovarian cancer in her 40's, one paternal uncle had colon cancer in his 40's, her paternal cousin has breast cancer in her 40's, and her father had colon cancer in his 40's. She denies cancer on her mother's side of the family. Bernice states she has 2 younger sisters. Which of the following responses would be best for the nurse to say to Bernice about her reported family history? Select one: a. "Do you have cancer on your mother's side of the family?" b. "Have you had your pap smear recently?" c. "The types of cancers reported on your father's side of the family may be due to a gene mutation that can be passed onto to each generation. Have any of your relatives or you considered genetic testing?" d. "Since all of these cancers are on your father's side of the family, you have no increased risk for cancer." e. "Have you had a screening mammogram?"

c. "The types of cancers reported on your father's side of the family may be due to a gene mutation that can be passed onto to each generation. Have any of your relatives or you considered genetic testing?"

A 35-year-old G1 P1 postpartum woman is Rh negative and has given birth to an Rh positive infant. Rh0(D) immune globulin is to be administered. The most appropriate dose that the perinatal nurse would expect to be ordered would be: Select one: a. 120 mcg b. 250 mcg c. 300 mcg d. 350 mcg

c. 300 mcg Nonsensitized women who are Rh negative and have given birth to an Rh positive infant should receive 300 mcg of Rh(D) immune globulin (RhoGAM) within 72 hours after giving birth. RhoGAM should be given whether or not the mother received RhoGAM during the antepartum period. In some situations, depending on the extent of hemorrhage and exchange of maternal-fetal blood, a larger dose of RhoGAM may be indicated.

Mary G10000 is 38 weeks pregnant and has been diagnosed with severe preeclampsia. The physician orders magnesium sulfate infusion for Mary, which includes a maintenance dose of 2 grams/hr. Her IV is a premixed magnesium sulfate solution of 40grams per 1000mL. What will the nurse program the infusion pump to deliver 2 grams/hr? Select one: a. 20mL/hr b. 80mL/hr c. 50 mL/hr d. 40mL/hr

c. 50 mL/hr 40 gm = 40,000mg 40,000mg:1000mL:: X: 1mL 1000X = 40000 X= 40mg 2 grams = 2000 mg 2000mg: X:: 40mg: 1 mL X=50mL/hr

Necrotizing enterocolitis (NEC) is an inflammatory disease of the gastrointestinal mucosa. The signs of NEC are nonspecific. What are generalized signs and symptoms of this condition? a.Scaphoid abdomen, no residual with feedings, and increased urinary output b. Hypertension, absence of apnea, and ruddy skin color c. Abdominal distention, temperature instability, and grossly bloody stools d. Hypertonia, tachycardia, and metabolic alkalosis

c. Abdominal distention, temperature instability, and grossly bloody stools

The nurse completes an initial newborn examination on a baby boy at 90 minutes of age. The baby was born at 40 weeks' gestation with no birth trauma. The nurse's findings include the following parameters: heart rate 136 beats per minute; respiratory rate 64 breaths per minute; temperature 98.2°F (36.8°C); length 49.5 cm; and weight 3500 g. The nurse documents the presence of a heart murmur, absence of bowel sounds, symmetry of ears and eyes, no grunting or nasal flaring, and full range of movement of all extremities. Which assessment would warrant further investigation and require immediate consultation with the baby's health-care provider? Select one: a. Respiratory rate b. Presence of a heart murmur c. Absent bowel sounds. d. Weight

c. Absent bowel sounds.

Approximately 8 hours ago, Juanita, a 32-year-old G1 P0, gave birth after 2 hours of pushing. She required an episiotomy and an assisted birth (forceps) due to the weight and size of her baby (9 lb. 9 oz.). The perinatal nurse is performing an assessment of Juanita's perineal area. A slight bulge is palpated and the presence of ecchymoses to the right of the episiotomy is noted. The area feels "full" and is approximately 4 cm in diameter. Juanita describes this area as "very tender." The perinatal nurse notifies the physician of the findings related to Juanita's assessment. The first step in care will most likely be to: Select one: a. Prepare Juanita for surgery b. Administer intravenous fluids c. Apply ice to the perineum d. Insert a urinary catheter

c. Apply ice to the perineum

A 3.8-kg infant was vaginally delivered at 39 weeks after a 30-minute second stage. A nuchal cord was found at delivery. After birth, the infant is noted to have petechiae over the face and upper back. Which information regarding petechiae is most accurate and should be provided to the parents? Select one: a. Result from increased blood volume b. Should always be further investigated c. Are benign if they disappear within 48 hours of birth d. Usually occur with a forceps-assisted delivery

c. Are benign if they disappear within 48 hours of birth

What is the highest priority nursing intervention when admitting a pregnant woman who has experienced a bleeding episode in late pregnancy? Select one: a. Performing a venipuncture for hemoglobin and hematocrit levels b. Monitoring uterine contractions c. Assessing FHR and maternal vital signs d. Placing clean disposable pads to collect any drainage

c. Assessing FHR and maternal vital signs

Which nursing intervention should be immediately performed after the forceps-assisted birth of an infant? Select one: a. Administering prophylactic antibiotic agents to the infant b. Measuring the circumference of the infant's head c. Assessing the infant for signs of trauma d. Applying a cold pack to the infant's scalp

c. Assessing the infant for signs of trauma

Preconception counseling is critical in the safe management of diabetic pregnancies. Which complication is commonly associated with poor glycemic control before and during early pregnancy? Select one: a. Hydramnios b. Hyperemesis gravidarum c. Congenital anomalies in the fetus d. Frequent episodes of maternal hypoglycemia

c. Congenital anomalies in the fetus

A postpartum client is concerned that her breasts are engorged and uncomfortable. What is the nurse's explanation for this physiologic change? Select one: a. Overproduction of colostrum b. Hyperplasia of mammary tissue c. Congestion of veins and lymphatic vessels d. Accumulation of milk in the lactiferous ducts and glands

c. Congestion of veins and lymphatic vessels

A postpartum woman telephones the provider regarding her 5-day-old infant. The client is not scheduled for another weight check until the infant is 14 days old. The new mother is worried about whether breastfeeding is going well. Which statement indicates that breastfeeding is effective for meeting the infant's nutritional needs? Select one: a. Sleeps for 6 hours at a time between feedings b. Gains 1 to 2 ounces per week c. Has at least six to eight wet diapers per day d. Has at least one breast milk stool every 24 hours

c. Has at least six to eight wet diapers per day

Primary prevention of preterm labor and birth include which of the following? (Select all that apply). Select one or more: a. Induce pregnant woman at 37 weeks b. Treat pregnant woman with magnesium sulfate c. Identify risk factors of preterm birth for individual pregnant woman d. Manage risk factors for preterm birth in individual pregnant woman

c. Identify risk factors of preterm birth for individual pregnant woman d. Manage risk factors for preterm birth in individual pregnant woman

Which important component of nutritional counseling should the nurse include in health teaching for a pregnant woman who is experiencing cholecystitis? Select one: a. Assess the woman's dietary history for adequate calories and proteins. b. Instruct the woman to eat a low-cholesterol, low-salt diet. c. Instruct the woman to eat a low-fat diet and to avoid fried foods d. Teach the woman that the bulk of calories should come from proteins.

c. Instruct the woman to eat a low-fat diet and to avoid fried foods

The nurse who is caring for a woman hospitalized for hyperemesis gravidarum would expect the initial treatment to involve what? Select one: a. Corticosteroids to reduce inflammation b. Antiemetic medication, such as pyridoxine, to control nausea and vomiting c. Intravenous (IV) therapy to correct fluid and electrolyte imbalances d. Enteral nutrition to correct nutritional deficits

c. Intravenous (IV) therapy to correct fluid and electrolyte imbalances

Which statement most accurately describes complicated grief? Select one: a. Is felt by the family of adolescent mothers who lose their babies b. Occurs when, in multiple births, one child dies and the other or others live c. Is an extremely intense grief reaction that persists for a long time d. Is a state during which the parents are ambivalent, as with an abortion

c. Is an extremely intense grief reaction that persists for a long time

18 year old Ellen has a positive pregnancy test and cries when she sees the result. Per her LMP, the nurse determines she is approx 7 weeks pregnant. She asks the nurse what should she do. Which of the following is the nurse's best initial response? Select one: a. I think adoption would be your best choice at this time. b. You need to decide what obstetrician you want to see for prenatal care. c. Let's discuss your three options. d. Here is a list of clinics that provide abortions. My sister liked this one best.

c. Let's discuss your three options.

According to the CDC, which of the following are true? Select one: a. Maternal mortality rate in the US has increased but infection is no longer a significant cause. b. Maternal mortality rate in the US has decreased from 1987-2013 c. Maternal mortality rate in the US has increased from 1987-2013 d. Maternal mortality rate in the US has increased primarily due to anesthesia complications

c. Maternal mortality rate in the US has increased from 1987-2013

By understanding the four mechanisms of heat transfer (convection, conduction, radiation, and evaporation), the nurse can create an environment for the infant that prevents temperature instability. Which significant symptoms will the infant display when experiencing cold stress? Select one: a. Increased physical activity b. Bradycardia, followed by an increased heart rate c. Mottled skin with acrocyanosis d. Decreased respiratory rate

c. Mottled skin with acrocyanosis

The nurse is completing the lab order for a pap smear performed on a 22 year old female. Which of the following options will the nurse select? Select one: a. Pap smear- cytology with reflux to high risk HPV test for ASCUS result b. Pap smear - no cytology, high risk HPV testing only c. Pap smear - cytology only d. Pap smear- cytology with high risk HPV test

c. Pap smear - cytology only

What marks on a baby's skin may indicate an underlying problem that requires notification of a physician? Select one: a. Mongolian spots on the back b. Telangiectatic nevi on the nose or nape of the neck c. Petechiae scattered over the infant's body d. Erythema toxicum neonatorum anywhere on the body

c. Petechiae scattered over the infant's body

During a telephone follow-up conversation with a woman who is 4 days postpartum, the woman tells the nurse, "I don't know what's wrong. I love my son, but I feel so let down. I seem to cry for no reason!" Which condition might this new mother be experiencing? Select one: a. Attachment difficulty b. Postpartum depression (PPD) c. Postpartum blues d. Letting-go

c. Postpartum blues

A woman who is 12 weeks postpartum presents with the following behavior: she reports severe mood swings and hearing voices, believes her infant is going to die, she has to be reminded to shower and put on clean clothes, and she feels she is unable to care for her baby. These behaviors are associated with which of the following? Select one: a. Postpartum blues b. Postpartum depression c. Postpartum psychosis d. Maladaptive mother-infant attachment

c. Postpartum psychosis (Postpartum psychosis (PPP) is a variant of bipolar disorder and is the most serious form of postpartum mood disorders. Onset of symptoms can be as early as the 3rd postpartum day. Assessment findings include paranoia, delusions associated with the baby, mood swings, extreme agitation, confused thinking, inability to care for self or infant, and strange beliefs.)

While providing care to the maternity client, the nurse should be aware that one of these anxiety disorders is likely to be triggered by the process of labor and birth. Which disorder fits this criterion? Select one: a. Panic disorder b. Obsessive-compulsive disorder (OCD) c. Posttraumatic stress disorder (PTSD) d. Phobias

c. Posttraumatic stress disorder (PTSD)

The perinatal nurse is providing care to Carol, a 28-year-old multiparous woman in labor. Upon arrival to the birthing suite, Carol was 7 cm dilated and experiencing contractions every 1 to 2 minutes which she describes as "strong." Carol states she labored for 1 hour at home. As the nurse assists Carol from the assessment area to her labor and birth room, Carol states that she is feeling some rectal pressure. Carol is most likely experiencing: Select one: a. Hypertonic contractions b. Hypotonic contractions c. Precipitous labor d. Uterine hyperstimulation

c. Precipitous labor

The nurse who elects to work in the specialty of obstetric care must have the ability to distinguish between preterm birth, preterm labor, and low birth weight. Which statement regarding this terminology is correct? Select one: a. Low birth weight is a newborn who weighs below 3.7 pounds. b. Terms preterm birth and low birth weight can be used interchangeably. c. Preterm labor is defined as cervical changes and uterine contractions occurring between 20 and 37 weeks of gestation. d. Preterm birth rate in the United States continues to increase

c. Preterm labor is defined as cervical changes and uterine contractions occurring between 20 and 37 weeks of gestation.

The nurse should be cognizant of which postpartum physiologic alteration? Select one: a. Hypercoagulable state protects the new mother from thromboembolism, especially after a cesarean birth. b. Cardiac output, pulse rate, and stroke volume all return to prepregnancy normal values within a few hours of childbirth. c. Respiratory function returns to nonpregnant levels by 6 to 8 weeks after childbirth d. Lowered white blood cell count after pregnancy can lead to false-positive results on tests for infections.

c. Respiratory function returns to nonpregnant levels by 6 to 8 weeks after childbirth

A client with maternal phenylketonuria (PKU) has come to the obstetrical clinic to begin prenatal care. Why would this preexisting condition result in the need for closer monitoring during pregnancy? Select one: a. A pregnant woman is more likely to die without strict dietary control. b. Women with PKU are usually mentally handicapped and should not reproduce. c. The fetus may develop neurologic problems. d. PKU is a recognized cause of preterm labor.

c. The fetus may develop neurologic problems.

Another common pregnancy-specific condition is pruritic urticarial papules and plaques of pregnancy (PUPPP). A client asks the nurse why she has developed this condition and what can be done. What is the nurse's best response? a.This common pregnancy-specific condition is associated with a poor fetal outcome. b. PUPPP is associated with decreased maternal weight gain. c. The goal of therapy is to relieve discomfort. d. The rate of hypertension decreases with PUPPP

c. The goal of therapy is to relieve discomfort.

The nurse is about to elicit the Moro or Startle reflex. Which of the following responses should the nurse expect to see? Select one: a. When the cheek of the baby is touched, the newborn turns toward the side that is touched. b. When the lateral aspect of the sole of the baby's foot is stroked, the toes extend and fan outward. c. When the baby is suddenly lowered or startled, the neonate's arms straighten outward and the knees flex. d. When the newborn is supine and the head is turned to one side, the arm on that same side extends.

c. When the baby is suddenly lowered or startled, the neonate's arms straighten outward and the knees flex. -The Moro or Startle Reflex is elicited when the neonate is suddenly lowered or exposed to a loud noise: symmetrical abduction and extension of the arms and legs with legs flexed up against the trunk. Choice 1 indicates the Rooting Reflex; Choice 2 indicates the Babinski Reflex; Choice 4 indicates the Tonic Neck Reflex

Which client is at greatest risk for early PPH? Select one: a. Primiparous woman (G 2, P 1-0-0-1) being prepared for an emergency cesarean birth for fetal distress b. Multiparous woman (G 3, P 2-0-0-2) with an 8-hour labor c. Woman with severe preeclampsia on magnesium sulfate whose labor is being induced d. Primigravida in spontaneous labor with preterm twins

c. Woman with severe preeclampsia on magnesium sulfate whose labor is being induced

Which of the following is the most effective contraception? Select one: a. the vaginal ring b. condoms c. the implant d. oral contraceptive pills e. natural family planning

c. the implant

16 year old Emily lives in Blackfoot, ID. She has a boyfriend but does not want to have a baby for at least 10 years. She wants an effective, easy birth control method and denies medical problems. She has periods every 4-6 weeks. The nurse would recommend which of the following methods to Emily? (Select all that apply). Select one or more: a. natural family planning b. bilateral tubal ligation c. the implant d. hormonal IUD

c. the implant d. hormonal IUD

A neonate born at 36 weeks gestation is classified as which of the following? a. Very preterm b.Moderately preterm c.Late preterm d.Term

c.Late preterm

During labor induction with oxytocin, the fetal heart rate baseline is in the 140s with moderate variability. Contraction frequency is assessed to be every 2-3 minutes with duration of 60 seconds, of moderate strength to palpation. Based on this assessment, the nurse should take which action? a.Increase oxytocin infusion rate per provider's protocol b.Stop oxytocin infusion immediately c.Maintain present oxytocin infusion rate and continue to assess. d.Decrease oxytocin infusion rate by 2 mU/min and report to provider.

c.Maintain present oxytocin infusion rate and continue to assess.

Which of these medications is commonly used to control postpartum bleeding related to uterine atony? a.Magnesium sulfate b.Phytonadione c.Oxytocin d.Warfarin

c.Oxytocin

A multipara, 26 weeks' gestation and accompanied by her husband, has just delivered a fetal demise. Which of the following nursing actions is appropriate at this time? a.Encourage the parents to pray for the baby's soul. b.Advise the parents that it is better for the baby to have died than to have had to live with a defect. c.Provide opportunities for grieving parents and family members to spend time with the baby d.Advise the parents to refrain from discussing the baby's death with their other children.

c.Provide opportunities for grieving parents and family members to spend time with the baby

A family is visiting two surviving triplets. The third triplet died 2 days ago. What action indicates that the family has begun to grieve for the dead infant? a.Refers to the two live infants as twins b.Asks about the dead triplet's current status c.Refers to the dead infant in the past tense d.Brings in play clothes for all three infants

c.Refers to the dead infant in the past tense

Jennifer is a 32 year old lawyer 37 weeks pregnant with her first child. She tells you that she has been on Paroxetine (an SSRI) for anxiety throughout her pregnancy. She asks you if she needs to worry about any side effects for her baby. Your best response would be: a.Yes, taking SSRIs in late pregnancy can cause symptoms in the baby but these won't show up for several months so I'll share with you what to look for before you are discharged. b.Yes, taking SSRIs in late pregnancy can be dangerous to the baby. You need to stop taking your Paroxetine immediately. c.SSRIs have been linked to symptoms in the newborn and while they don't usually last too long I will alert your pediatrician and our high-risk nursery so everyone is aware. d.You probably don't need to worry as the use of SSRIs in pregnancy is not a problem.

c.SSRIs have been linked to symptoms in the newborn and while they don't usually last too long I will alert your pediatrician and our high-risk nursery so everyone is aware.

Human immunodeficiency virus (HIV) may be transmitted perinatally or during the postpartum period. Which statement regarding the method of transmission is most accurate? a.From the use of unsterile instruments b.Only in the third trimester from the maternal circulation c.Through the ingestion of breast milk from an infected mother d.Only through the ingestion of amniotic fluid

c.Through the ingestion of breast milk from an infected mother

A 25-year-old multiparous woman gave birth to an infant boy 1 day ago. Today her husband brings a large container of brown seaweed soup to the hospital. When the nurse enters the room, the husband asks for help with warming the soup so that his wife can eat it. What is the nurse's most appropriate response? Select one: a. "What is that anyway?" b. "Does your physician know that you are planning to eat that?" c. "Didn't you like your lunch?" d. "I'll warm the soup in the microwave for you."

d. "I'll warm the soup in the microwave for you."

A perinatal nurse is giving discharge instructions to a woman, status postsuction, and curettage secondary to a hydatidiform mole. The woman asks why she must take oral contraceptives for the next 12 months. What is the best response by the nurse? Select one: a. "If you get pregnant within 1 year, the chance of a successful pregnancy is very small. Therefore, if you desire a future pregnancy, it would be better for you to use the most reliable method of contraception available." b. "If you can avoid a pregnancy for the next year, the chance of developing a second molar pregnancy is rare. Therefore, to improve your chance of a successful pregnancy, not getting pregnant at this time is best." c. "Oral contraceptives are the only form of birth control that will prevent a recurrence of a molar pregnancy." d. "The major risk to you after a molar pregnancy is a type of cancer that can be diagnosed only by measuring the same hormone that your body produces during pregnancy. If you were to get pregnant, then it would make the diagnosis of this cancer more difficult."

d. "The major risk to you after a molar pregnancy is a type of cancer that can be diagnosed only by measuring the same hormone that your body produces during pregnancy. If you were to get pregnant, then it would make the diagnosis of this cancer more difficult."

A healthy 60-year-old African-American woman regularly receives health care at her neighborhood clinic. She is due for a mammogram. At her first visit, her health care provider is concerned about the 3-week wait at the neighborhood clinic and made an appointment for her to have a mammogram at a teaching hospital across town. She did not keep her appointment and returned to the clinic today to have the nurse check her blood pressure. What is the most appropriate statement for the nurse to make to this client? Select one: a. "I'm concerned that you missed your appointment; let me make another one for you." b. "Do you have transportation to the teaching hospital so that you can get your mammogram?" c. "It's very dangerous to skip your mammograms; your breasts need to be checked." d. "Would you like me to make an appointment for you to have your mammogram here?"

d. "Would you like me to make an appointment for you to have your mammogram here?" The BEST answer to meet this pt's needs. It is evident the pt can make it to the neighborhood clinic, not the hospital across town.

The clinic nurse sees Xiao and her infant in the clinic for their 2-week follow-up visit. Xiao appears to be tired, her clothes and hair appear unwashed, and she does not make eye contact with her infant. She is carrying her son in the infant carrier and when asked to put him on the examining table, she holds him away from her body. The clinic nurse's most appropriate question to ask would be: Select one: a. "What has happened to you?" b. "Do you have help at home?" c. "Is there anything wrong with your son?" d. "Would you tell me about the first few days at home?"

d. "Would you tell me about the first few days at home?"

Which woman is at the greatest risk for psychologic complications after hysterectomy? Select one: a. 62-year-old widow who has three friends who have had uncomplicated hysterectomies b. 46-year-old woman who has had three children and has just been promoted at work c. 55-year-old woman who has been having abnormal bleeding and pain for 3 years d. 19-year-old woman who had a ruptured uterus after giving birth to her first child

d. 19-year-old woman who had a ruptured uterus after giving birth to her first child

A postmenopausal woman has been diagnosed with two leiomyomas (fibroids). Which clinical finding is most commonly associated with the presence of leiomyomas? Select one: a. Diarrhea b. Acute abdominal pain c. Weight loss d. Abnormal uterine bleeding

d. Abnormal uterine bleeding

Which substance, when abused during pregnancy, is the most significant cause of cognitive impairment and dysfunction in the infant? Select one: a. Tobacco b. Heroin c. Marijuana d. Alcohol

d. Alcohol

The obstetric provider has informed the nurse that she will be performing an amniotomy on the client to induce labor. What is the nurse's highest priority intervention after the amniotomy is performed? Select one: a. Applying clean linens under the woman b. Performing a vaginal examination c. Taking the client's vital signs d. Assessing the fetal heart rate (FHR)

d. Assessing the fetal heart rate (FHR)

The symptoms of mild to moderate urinary incontinence can be successfully decreased by a number of strategies. Which of these should the nurse instruct the client to use first? Select one: a. Surgery b. Medications c. Pelvic floor support devices d. Bladder training and pelvic muscle exercises or Kegels

d. Bladder training and pelvic muscle exercises or Kegels

As a powerful central nervous system (CNS) stimulant, which of these substances can lead to miscarriage, preterm labor, placental separation (abruption), and stillbirth? Select one: a. Heroin b. Alcohol c. Phencyclidine (1-phenylcyclohexylpiperidine; PCP) d. Cocaine

d. Cocaine

A new mother asks whether she should feed her newborn colostrum, because it is not "real milk." What is the nurse's most appropriate answer? Select one: a. Colostrum is unnecessary for newborns. b. Colostrum is lower in calories than milk and should be supplemented by formula. c. Giving colostrum is important in helping the mother learn how to breastfeed before she goes home. d. Colostrum is high in antibodies, protein, vitamins, and minerals.

d. Colostrum is high in antibodies, protein, vitamins, and minerals.

Which clinical findings would alert the nurse that the neonate is expressing pain? Select one: a. High-pitched, shrill cry; withdrawal; change in heart rate b. Low-pitched crying; tachycardia; eyelids open wide c. Cry face; flaccid limbs; closed mouth d. Cry face; eyes squeezed; increase in blood pressure

d. Cry face; eyes squeezed; increase in blood pressure

A client in late middle age who is certain she is not pregnant tells the nurse during an office visit that she has urinary problems, as well as sensations of bearing down and of something in her vagina. What condition would the nurse suspect based upon this report? Select one: a. Genital fistulas b. Pelvic relaxation c. Uterine prolapse d. Cystocele and/or rectocele

d. Cystocele and/or rectocele

An 18-year-old client who has reached 16 weeks of gestation was recently diagnosed with pregestational diabetes. She attends her centering appointment accompanied by one of her girlfriends. This young woman appears more concerned about how her pregnancy will affect her social life than her recent diagnosis of diabetes. A number of nursing diagnoses are applicable to assist in planning adequate care. What is the most appropriate diagnosis at this time? Select one: a. Deficient knowledge, related to insulin administration b. Risk for injury, to the fetus related to birth trauma c. Risk for injury, to the mother related to hypoglycemia or hyperglycemia d. Deficient knowledge, related to diabetic pregnancy management

d. Deficient knowledge, related to diabetic pregnancy management

The perinatal nurse listens as Chantal describes her labor and emergency cesarean birth. Providing an opportunity to review this experience may assist Chantal in: Select one: a. Her role development in the "letting go" stage b. Decreasing her ambivalence about her labor and birth c. Understanding her guilt involved in her labor and birth d. Developing more positive feelings about her labor and birth

d. Developing more positive feelings about her labor and birth

Which statement concerning the complication of maternal diabetes is the most accurate? Select one: a. Infections occur about as often and are considered about as serious in both diabetic and nondiabetic pregnancies. b. Hydramnios occurs approximately twice as often in diabetic pregnancies than in nondiabetic pregnancies. c. Even mild-to-moderate hypoglycemic episodes can have significant effects on fetal well-being. d. Diabetic ketoacidosis (DKA) can lead to fetal death at any time during pregnancy

d. Diabetic ketoacidosis (DKA) can lead to fetal death at any time during pregnancy

Which of the following is true regarding dizygotic twins? Select one: a. The fetuses share one chorion. b. Each fetus has their own uterus. c. The fetuses share one amnion. d. Each fetus has their own placenta.

d. Each fetus has their own placenta.

The perinatal nurse teaches the postpartum woman about the normal process of diuresis that she can expect to occur approximately 6 to 8 hours after birth. A decrease in which of the following hormones is primarily responsible for the diuresis? Select one: a. Prolactin b. Progesterone c. Lactogen d. Estrogen

d. Estrogen

What information is important for the nurse to include in planning for the care of a woman who has had a vaginal hysterectomy (no oopherectomy) for fibroids? Select one: a. Expect surgical menopause. b. Expect to be fully recovered in 4 to 6 weeks. c. Next pap smear is due in one year. d. Expect no changes in her hormone levels

d. Expect no changes in her hormone levels

What is the most important nursing action in preventing neonatal infection? a.isolation of infected infants b. standard precautions c.Speratate gown technique d. Good handwashing or hand hygiene.

d. Good handwashing or hand hygiene.

When a woman is diagnosed with postpartum depression (PPD) with psychotic features, what is the nurse's primary concern in planning the client's care? Select one: a. Losing interest in her husband b. Displaying outbursts of anger c. Neglecting her hygiene d. Harming her infant

d. Harming her infant

Which condition is considered a medical emergency that requires immediate treatment? Select one: a. Uterine atony b. Hypotonic uterus c. ITP d. Inversion of the uterus

d. Inversion of the uterus

Which of the following is true regarding physiologic jaundice? (Select all that apply). Select one or more: a. Jaundice persists longer than 14 days of life in a term infant b. Jaundice is noted within 24 hours after birth c. It is recommended the mother stop breastfeeding until jaundice resolves d. Jaundice is more common in preterm infants e. Jaundice is noted 24 hours after birth

d. Jaundice is more common in preterm infants e. Jaundice is noted 24 hours after birth

On examining a woman who gave birth 5 hours ago, the nurse finds that the woman has completely saturated a perineal pad within 15 minutes. What is the nurse's highest priority at this time? Select one: a. Calling the woman's primary health care provider b. Beginning an intravenous (IV) infusion of Ringer's lactate solution c. Assessing the woman's vital signs d. Massaging the woman's fundus

d. Massaging the woman's fundus

The nurse has evaluated a client with preeclampsia by assessing DTRs. The result is a grade of 3+. Which DTR response most accurately describes this score? Select one: a. Active or expected response b. Sluggish or diminished c. Brisk, hyperactive, with intermittent or transient clonus d. More brisk than expected, slightly hyperactive

d. More brisk than expected, slightly hyperactive

A nurse assesses that a 3-day-old neonate who was born at 34 weeks' gestation has abdominal distention and vomiting. These assessment findings are most likely related to: Select one: a. Respiratory Distress Syndrome (RDS) b. Bronchopulmonary Dysplasia (BPD) c. Periventricular Hemorrhage (PVH) d. Necrotizing Enterocolitis (NEC)

d. Necrotizing Enterocolitis (NEC)

A nursery nurse observes that a full-term AGA neonate has nasal congestion, hypertonia, and tremors and is extremely irritable. Based on these observations, the nurse suspects which of the following? Select one: a. Hypoglycemia b. Hypercalcemia c. Cold stress d. Neonatal withdrawal

d. Neonatal withdrawal

If the umbilical cord prolapses during labor, the nurse should immediately: Select one: a. Type and cross-match blood for an emergency transfusion. b. Await provider order for preparation for an emergency cesarean section. c. Attempt to reposition the cord above the presenting part. d. Perform vaginal exam and lifting the presenting part off of the cord to relieve pressure on the cord.

d. Perform vaginal exam and lifting the presenting part off of the cord to relieve pressure on the cord.

What is the primary nursing responsibility when caring for a client who is experiencing an obstetric hemorrhage associated with uterine atony? Select one: a. Establishing venous access b. Catheterizing the bladder c. Preparing the woman for surgical intervention d. Performing fundal massage

d. Performing fundal massage

A first-time dad is concerned that his 3-day-old daughter's skin looks "yellow." In the nurse's explanation of physiologic jaundice, what fact should be included? Select one: a. Physiologic jaundice is also known as breast milk jaundice. b. Physiologic jaundice occurs during the first 24 hours of life. c. Physiologic jaundice is caused by blood incompatibilities between the mother and the infant blood types. d. Physiologic jaundice becomes visible when serum bilirubin levels peak between the second and fourth days of life

d. Physiologic jaundice becomes visible when serum bilirubin levels peak between the second and fourth days of life

Parents have been asked by the neonatologist to provide breast milk for their newborn son, who was born prematurely at 32 weeks of gestation. The nurse who instructs them regarding pumping, storing, and transporting the milk needs to assess their knowledge of lactation. Which statement is valid? Select one: a. A glass of wine just before pumping will help reduce stress and anxiety. b. The mother should only pump as much milk as the infant can drink. c. The mother should pump every 2 to 3 hours, including during the night. d. Premature infants more easily digest breast milk than formula.

d. Premature infants more easily digest breast milk than formula.

According to research, which risk factor for PPD is likely to have the greatest effect on the client postpartum? Select one: a. Low socioeconomic status b. Single-mother status c. Unplanned or unwanted pregnancy d. Prenatal depression

d. Prenatal depression

After giving birth to a healthy infant boy, a primiparous client, 16 years of age, is admitted to the postpartum unit. An appropriate nursing diagnosis for her at this time is "Deficient knowledge of infant care." What should the nurse be certain to include in the plan of care as he or she prepares the client for discharge? Select one: a. Teach the client how to feed and bathe her infant. b. Give the client written information on bathing her infant. c. Advise the client that all mothers instinctively know how to care for their infants. d. Provide time for the client to bathe her infant after she views a demonstration of infant bathing.

d. Provide time for the client to bathe her infant after she views a demonstration of infant bathing.

Because of the premature infant's decreased immune functioning, what nursing diagnosis should the nurse include in a plan of care for a premature infant? a. Ineffective thermoregulation b. Delayed growth and development c. Ineffective infant feeding pattern d. Risk for infection

d. Risk for infection

The laboratory results for a postpartum woman are as follows: blood type, A; Rh status, positive; rubella non-immune (titer 1:8 or enzyme immunoassay [EIA] 0.8); hematocrit, 30%. How should the nurse best interpret these data? Select one: a. Kleihauer-Betke test should be performed. b. Rh immune globulin is necessary within 72 hours of childbirth. c. Blood transfusion is necessary. d. Rubella vaccine should be administered.

d. Rubella vaccine should be administered.

The most effective and least expensive treatment of puerperal infection is prevention. What is the most important strategy for the nurse to adopt? Select one: a. Large doses of vitamin C during pregnancy b. Prophylactic antibiotics c. Limited protein and fat intake d. Strict aseptic technique, including hand washing, by all health care personnel

d. Strict aseptic technique, including hand washing, by all health care personnel

A woman gave birth to a 7-pound, 6-ounce infant girl 1 hour ago. The birth was vaginal and the estimated blood loss (EBL) was 1500 ml. When evaluating the woman's vital signs, which finding would be of greatest concern to the nurse? Select one: a. Temperature 36.8° C, heart rate 60 bpm, respirations 18 breaths per minute, and blood pressure 140/90 mm Hg b. Temperature 38° C, heart rate 80 bpm, respirations 16 breaths per minute, and blood pressure 110/80 mm Hg c. Temperature 37.4° C, heart rate 88 bpm, respirations 36 breaths per minute, and blood pressure 126/68 mm Hg d. Temperature 37.9° C, heart rate 120 beats per minute (bpm), respirations 20 breaths per minute, and blood pressure 90/50 mm Hg

d. Temperature 37.9° C, heart rate 120 beats per minute (bpm), respirations 20 breaths per minute, and blood pressure 90/50 mm Hg

In follow-up appointments or visits with parents and their new baby, it is useful if the nurse can identify infant behaviors that can either facilitate or inhibit attachment. What is an inhibiting behavior? Select one: a. The infant clings to the parents. b. The infant cries only when hungry or wet. c. The infant's activity is somewhat predictable. d. The infant seeks attention from any adult in the room.

d. The infant seeks attention from any adult in the room.

A newly delivered mother who intends to breastfeed tells her nurse, "I am so relieved that this pregnancy is over so that I can start smoking again." The nurse encourages the client to refrain from smoking. However, this new mother is insistent that she will resume smoking. How will the nurse adapt her health teaching with this new information? Select one: a. Smoking has little-to-no effect on milk production b. The effects of secondhand smoke on infants are less significant than for adults. c. No relationship exists between smoking and the time of feedings. d. The mother should not smoke in the same room her baby is in.

d. The mother should not smoke in the same room her baby is in.

Which finding would indicate to the nurse that the grieving parents have progressed to the reorganization phase of grieving? Select one: a. The parents say that they "feel no pain." b. The parents' questions have progressed from "Why?" to "Why us?" c. The parents have abandoned those moments of "bittersweet grief." d. The parents are discussing sex and a future pregnancy, even if they have not yet sorted out their feelings.

d. The parents are discussing sex and a future pregnancy, even if they have not yet sorted out their feelings.

Which statement regarding the laboratory test for glycosylated hemoglobin Alc is correct? Select one: a. This laboratory test is a snapshot of glucose control at the moment. b. This laboratory test is performed on the woman's urine, not her blood. c. The laboratory test for glycosylated hemoglobin Alc is performed for all pregnant women, not only those with or likely to have diabetes. d. This laboratory test measures the levels of hemoglobin Alc, which should remain at less than 7%

d. This laboratory test measures the levels of hemoglobin Alc, which should remain at less than 7%

A new father wants to know what medication was put into his infant's eyes and why it is needed. How does the nurse explain the purpose of the erythromycin (Ilotycin) ophthalmic ointment? Select one: a. Erythromycin (Ilotycin) prevents potentially harmful exudate from invading the tear ducts of the infant's eyes, leading to dry eyes. b. This ointment prevents the infant's eyelids from sticking together and helps the infant see. c. Erythromycin (Ilotycin) ophthalmic ointment destroys an infectious exudate caused by Staphylococcus that could make the infant blind. d. This ophthalmic ointment prevents gonorrheal infection of the infant's eyes, potentially acquired from the birth canal.

d. This ophthalmic ointment prevents gonorrheal infection of the infant's eyes, potentially acquired from the birth canal.

A pregnant woman has been receiving a magnesium sulfate infusion for treatment of severe preeclampsia for 24 hours. On assessment, the nurse finds the following vital signs: temperature 37.3° C, pulse rate 88 beats per minute, respiratory rate of 10 breaths per minute, BP 148/90 mm Hg, absent deep tendon reflexes (DTRs), and no ankle clonus. The client complains, "I'm so thirsty and warm." What is the nurse's immediate action? Select one: a. To administer oxygen b. To call for an immediate magnesium sulfate level c. Call the physician d. To discontinue the magnesium sulfate infusion

d. To discontinue the magnesium sulfate infusion

During the initial acute distress phase of grieving, parents still must make unexpected and unwanted decisions about funeral arrangements and even naming the baby. What is the nurse's role at this time? Select one: a. To encourage the grandparents to take over b. To take over as much as possible to relieve the pressure c. To leave them alone to work things out d. To ensure that the parents, themselves, approve the final decisions

d. To ensure that the parents, themselves, approve the final decisions

What is the primary purpose for magnesium sulfate administration for clients with preeclampsia and eclampsia? Select one: a. To improve patellar reflexes and increase respiratory efficiency b. To shorten the duration of labor c. To prevent a boggy uterus and lessen lochial flow d. To prevent convulsions

d. To prevent convulsions

The nurse is circulating during a cesarean birth of a preterm infant. The obstetrician requests that cord clamping be delayed. What is the rationale for this directive? Select one: a. To decrease total blood volume b. To reduce the risk for jaundice c. To improve the ability to fight infection d. To reduce the risk of intraventricular hemorrhage

d. To reduce the risk of intraventricular hemorrhage

Which substance used during pregnancy causes vasoconstriction and decreased placental perfusion, resulting in maternal and neonatal complications? Select one: a. Chocolate b. Alcohol c. Caffeine d. Tobacco

d. Tobacco

Nurses need to understand the basic definitions and incidence data regarding PPH. Which statement regarding this condition is most accurate? Select one: a. PPH is easy to recognize early; after all, the woman is bleeding. b. Traditionally, it takes more than 1000 ml of blood after vaginal birth and 2500 ml after cesarean birth to define the condition as PPH. c. If anything, nurses and physicians tend to overestimate the amount of blood loss. d. Traditionally, PPH has been classified as early PPH or late PPH with respect to birth

d. Traditionally, PPH has been classified as early PPH or late PPH with respect to birth

In assisting the breastfeeding mother to position the baby, which information regarding positioning is important for the nurse to keep in mind? Select one: a. Women with perineal pain and swelling prefer the modified cradle position. b. The cradle position is usually preferred by mothers who had a cesarean birth. c. While supporting the head, the mother should push gently on the occiput. d. Whatever the position used, the infant is "belly to belly" with the mother.

d. Whatever the position used, the infant is "belly to belly" with the mother.

Which of the following can provide contraception and protection against sexually transmitted infections? (Select all that apply). Select one or more: a. birth control pills b. intrauterine devices c. diaphragm d. condoms e. abstinence f. natural family planning

d. condoms e. abstinence

Depo provera, or The Shot, contains which of the following hormones? Select one: a. estrogen b. estrogen and progesterone c. neither estrogen or progesterone d. progesterone

d. progesterone

Sally is 34 weeks pregnant and has a biophysical profile score of 8. She asks the nurse what does that mean? The nurse's best response is: a. "Since you are more than 36 weeks, it is best to deliver your baby as she may have asphyxia. I will call your doctor." b." Your baby may be having some problems and your doctor may want to check to see if it is safe to deliver her soon. I will call your doctor." c."Your baby is in danger of dying and you will probably need to deliver ASAP. I will call your doctor." d."Your baby has low risk for having a problem having enough oxygen in her body and your doctor will probably want to do the test again soon. I will call your doctor. "

d."Your baby has low risk for having a problem having enough oxygen in her body and your doctor will probably want to do the test again soon. I will call your doctor. "

Which of the following is true about twin pregnancy? a.The cesarean section rate for women with a singleton pregnancy is more than that for women with a twin pregnancy. b.Pregnant women with twins should gain weight as recommended for women with a singleton pregnancy. c.There is no more risk of complications during a twin pregnancy as compared to a singleton pregnancy. d.Pregnant women with twins usually require more surveillance during their pregnancy compared to women with singleton pregnancy.

d.Pregnant women with twins usually require more surveillance during their pregnancy compared to women with singleton pregnancy.

Parents are often asked if they would like to have an autopsy performed on their infant. Nurses who are assisting parents with this decision should be aware of which information? a.Autopsies are usually covered by insurance. b. In the current litigious society, more autopsies are performed than in the past. c.Autopsies must be performed within a few hours after the infant's death. d.Some religions prohibit autopsy.

d.Some religions prohibit autopsy.

Which is true regarding breast cyst? Select one: a. It requires surgery for diagnosis b. Removal is the best treatment c. It is usually fixed and hard D. It can be tender or not

D. It can be tender or not

PPH may be sudden and result in rapid blood loss. The nurse must be alert to the symptoms of hemorrhage and hypovolemic shock and be prepared to act quickly to minimize blood loss. Astute assessment of the client's circulatory status can be performed with noninvasive monitoring. Match the type of noninvasive assessment that the nurse would perform with the appropriate clinical manifestation or body system.

Palpation → arterial pulses, Observation → presence or absence of anxiety, Inspection → skin color, temperature, and turgor, Auscultation → heart pulses, Measurement → pulse oximetry

The nurse is using the New Ballard Scale to determine the gestational age of a newborn. Which assessment finding is consistent with a gestational age of 40 weeks? Select one: a. Flexed posture b. Smooth, pink skin with visible veins c. Abundant lanugo d. Faint red marks on the soles of the feet

a. Flexed posture

The nurse assessing a newborn for heat loss is aware that nonshivering thermogenesis utilizes the newborn's stores of brown adipose tissue (BAT) to provide heat in the cold-stressed newborn. True False

True

The nurse is preparing to teach an antepartum client with GDM the correct method of administering an intermediate-acting insulin, such as neutral protamine Hagedorn (NPH), with a short-acting insulin (regular). In the correct order from 1 through 6, match the step number with the action needed to teach the client self-administration of this combination of insulin.

Wash hands. Check the insulin bottles for the expiration date. Gently rotate the insulin to mix it, and wipe the stopper. Inject air equal to the dose of NPH insulin into the vial, and remove the syringe Inject air equal to the dose of regular insulin into the vial, and withdraw the medication Without adding air, withdraw the correct dose of NPH insulin.

The parents of a newborn ask the nurse how much the newborn can see. The parents specifically want to know what type of visual stimuli they should provide for their newborn. What information provided by the nurse would be most useful to these new parents? Select one: a. "It's important to shield the newborn's eyes. Overhead lights help them see better." b. "Infants can track their parents' eyes and can distinguish patterns; they prefer complex patterns." c. "The infant's eyes must be protected. Infants enjoy looking at brightly colored stripes." d. "Infants can see very little until approximately 3 months of age."

b. "Infants can track their parents' eyes and can distinguish patterns; they prefer complex patterns."

After the Nurse Practitioner treats a 25 y/o's Bartholin cyst with incision and drainage procedure (I&D), the nurse is providing take-home instructions. The patient asks, "Will this happen again?" The nurse's best response is: Select one: a. "I cannot answer that question for you. Let me have the NP come and talk to you." b. "If you take your antibiotics and not get an infection, recurrence is rare. Let's talk about how to prevent infection." c. "No, since it is drained it won't come back." d. "Bartholin cysts can recur. There is another procedure that can be done to reduce risk for recurrence.Would you like to talk to the NP about it?"

d. "Bartholin cysts can recur. There is another procedure that can be done to reduce risk for recurrence.Would you like to talk to the NP about it?"


Conjuntos de estudio relacionados

Chapter 41 Elimination Questions

View Set

chem preview assignments march 27, 29, 31

View Set